Sie sind auf Seite 1von 60

TEST - 1

Total Marks :

200

Mark Scored :

110.67

1. As the rulers of the mahajanapadas of the Common Era were building huge forts maintaining big armies, they needed
more resources for which they collected taxes. Consider the following about it.
1. Taxes on crop were fixed at a certain share of the total produce.
2. Craft persons were made to pay taxes
3. Hunter gatherers and herders were exempt from taxes as it was not possible to enumerate them as a part of any
specific empire.
4. Trade was tax-free but a surcharge was levied on special category of goods.
Select the correct answer using the codes below.
A. 1 and 2 only
B. 1, 3 and 4 only
C. 2 and 3 only
D. 1, 2, 3 and 4
User Answer : A
Correct Answer : A
Answer Justification :
Justification: Statement 1: Taxes on crops were the most important. This was because most people were farmers.
Usually, the tax was fixed at 1/6th of what was produced. This was known as bhaga or a share.
Statement 2: These could have been in the form of labour. For example, a weaver or a smith may have had to
work for a day every month for the king.
Statement 3: Herders were also expected to pay taxes in the form of animals and animal produce. And hunters and
gatherers also had to provide forest produce to the raja. So, 3 is incorrect.
Statement 4: There were also taxes on goods that were bought and sold, through trade. So, 4 is also incorrect.
Q Source: Page 59: NCERT 6th History: Our Pasts-I

2. Sariska Tiger Reserve is known for


A. Being the first Tiger reserve in the World to have successfully relocated tigers.
B. Confluence of four rivers at a single location harbouring one of the richest biodiversity of predators in
India.
C. Successfully stopping poaching of tigers and other related species completely
D. Being the first Tiger Reserve to be named in CITES agreement
User Answer :
Correct Answer : A
Answer Justification :
Justification: In 2004, it was found that no tigers were left in Sariska reserve, which the Rajasthan Forest
Department largely attributed initially to migration, but later to poaching.
The matter was taken seriously, and in 2005, the Government of Rajasthan, in cooperation with the Government of
India and Wildlife Institute of India (WII), planned the re-introduction of tigers to Sariska from Ranthambore tiger
reserve.
It thus became the first tiger reserve in the world to have successfully relocated tigers.
Learning: This area was a hunting preserve of the Alwar state and it was declared a wildlife reserve in 1955.
The area of Sariska, being a part of the Aravalli Range, is rich in mineral resources, such as copper.
The reserve is also the location of several sites of historical importance such as the 16th-century Kankwadi
(C) Insights Active Learning. | All rights reserved.

www.insightsias.com

TEST - 1

Total Marks :

200

Mark Scored :

110.67

fort, originally built by Jai Singh II, located near the centre of the park.
Pandupol in the hills in the centre of the reserve is believed to be one of the retreats of Pandava.
Q Source: Page 63 (news reports): NCERT 6th Geography: The Earth Our Habitat

3. The mid-day Sun will NEVER be exactly overhead on which of the following locations?
1.
2.
3.
4.

Any latitude in Polar Regions.


Any latitude where equatorial vegetation is found.
Any longitude that touches poles.
Any longitude that passes through Scandinavian countries.
Select the correct answer using the codes below.
A. 1 and 3 only
B. 2, 3 and 4 only
C. 1, 3 and 4 only
D. 1 only
User Answer : D
Correct Answer : D
Answer Justification :
Concept: The mid-day sun is exactly overhead at least once a year on all latitudes in between the Tropic of Cancer
and the Tropic of Capricorn. This area, therefore, receives the maximum heat and is called the Torrid Zone.
The mid-day sun never shines overhead on any latitude beyond the Tropic of Cancer and the Tropic of Capricorn.
The angle of the suns rays goes on decreasing towards the poles.
Justification: Statement 1: As evident from the explanation above, 1 is clearly an answer.
Statement 2: Equatorial vegetation can be found near equator, where Suns rays are generally overhead.
Statement 3: Every longitude starts from a pole and meets the other pole. It is latitude that matters in distribution
of solar insolation, not the longitude.
Statement 4: Based on the same logic as above, 4 is also incorrect.
Q Source: Page 12: NCERT 6th Geography: The Earth Our Habitat

4. What is the difference between the animals Changthangi and Chiru?


A. Changthangi lives in hot and arid areas, whereas Chiru is adapted to live in semi-desert areas of cold high
mountains
B. Changthangi is an antelope, whereas Chiru is a primate.
C. Changthangi is famously reared for Pashmina wool, whereas Chiru is reared for getting Shahtoosh wool
D. Changthangi is poached for its horns, whereas Chiru is hunted for its musk which has great medicinal value
User Answer : C
Correct Answer : C
Answer Justification :
Justification:Option A: The Changthangi or Pashmina goat inhabits the plateaus in Tibet and neighbouring areas
of Ladakh in Jammu & Kashmir. They are raised for ultra-fine cashmere wool (pashmina).
Tibetan Antelope of Chiru is native to Tibet.
Option B: Chiru is an antelope, whereas Changthangi is a goat. Primate are apes, humans etc.
(C) Insights Active Learning. | All rights reserved.

www.insightsias.com

TEST - 1

Total Marks :

200

Mark Scored :

110.67

Option C-D: In recent years Chiru has become endangered due to poaching for their soft and warm wool which is
usually obtained after death. This wool is known as shahtoosh and is used to weave shawls . At present,
international trade in their products is strictly prohibited.
Q Source: Improvisation: Page 75: Geography NCERT 7th: Our Environment

5. Why 15th August was chosen as the Independence Date of India?


A. It was on this date that Indian Independence Bill was introduced in the British House of Commons and
passed.
B. Lord Mountbatten chose the date as he considered this date to be lucky.
C. It was on this date that the Lahore Session of the Congress passed the Purna Swaraj resolution.
D. On this date, Indias Constituent Assembly passed the Objectives Resolution granting freedom to India.
User Answer : B
Correct Answer : B
Answer Justification :
Justification & Learning:Option A: Indian Independence Bill was introduced in the British House of Commons
on July 4, 1947 and passed within a fortnight.
Option B: The date was chosen by Lord Mountbatten himself because he had considered this date to be lucky. It
was on this day during the World War II, that the Japanese Army surrendered to the allies.
Option C: In 1929, when Jawaharlal Nehru as Congress President gave the call for Poorna Swaraj or total
independence from British colonial rule, January 26 was chosen as the Independence Day. Later it was declared as
the Republic Day.
Q
Source:http://indianexpress.com/article/india/india-news-india/why-was-august-15-chosen-as-independence-day/

6. NASA's Fermi Mission is aimed at understanding


1. Earths geological structure.
2. Role of exo-planets in providing a habitable space on earth.
3. Search for signs of new laws of physics and what composes the mysterious Dark Matter.
Select the correct answer using the codes below.
A. 1 only
B. 2 and 3 only
C. 3 only
D. None of the above
User Answer :
Correct Answer : C
Answer Justification :
Justification: NASA's Fermi (formerly Gamma-ray Large Area Space Telescope, or GLAST) is a powerful space
observatory (from 2008) that has started to:
Explore the most extreme environments in the Universe, where nature harnesses energies far beyond
anything possible on Earth.
Search for signs of new laws of physics and what composes the mysterious Dark Matter.
Explain how black holes accelerate immense jets of material to nearly light speed.
Help crack the mysteries of the stupendously powerful explosions known as gamma-ray bursts.
Answer long-standing questions across a broad range of topics, including solar flares, pulsars and the origin
(C) Insights Active Learning. | All rights reserved.

www.insightsias.com

TEST - 1

Total Marks :

200

Mark Scored :

110.67

of cosmic rays.
Learning: Gamma rays are the highest-energy form of light, and the gamma-ray sky is spectacularly different
from the one we perceive with our own eyes.
Fermi data has enabled scientists to answer persistent questions across a broad range of topics, including
supermassive black-hole systems, pulsars, the origin of cosmic rays, and searches for signals of new physics.
Q Source: Current Affairs: NASA Website:
https://www.nasa.gov/feature/goddard/2016/nasas-fermi-mission-expands-its-search-for-dark-matter

7. Consider the following about spot markets in India.


1. They are electronic financial markets where on spot deals for futures in commodities are fixed.
2. National Spot Exchange Limited (NSEL) is the only recognized spot market in India.
3. They are regulated by the Securities and Exchange Board of India (SEBI).
Select the correct answer using the codes below.
A. 1 and 2 only
B. 1 and 3 only
C. 3 only
D. 1, 2 and 3
User Answer : D
Correct Answer : C
Answer Justification :
Justification: Statement 1: It is a public financial market in which financial instruments or commodities are
traded for immediate delivery. It contrasts with a futures market, in which delivery is due at a later date. So, 1 is
incorrect.
Statement 2: There are many players in the sport market, for e.g. NCDEX Spot Exchange Limited, Indian Bullion
Spot Exchange limited. NSEL is only one of them, which was also in news due to a scam.
Statement 3: Spot Exchange is presently recognized by Ministry of Consumer Affairs, Food & Public
Distribution. It was regulated by the Forward Contracts Act earlier by Forward Markets Commission. Now, since
FMC has been merged with SEBI, SEBI regulates it.
Learning: Current events section explains the NSEL scam very well
http://www.insightsonindia.com/2016/08/01/insights-daily-current-affairs-01-august-2016/
You can also go through this to know more about spot markets
http://www.arthapedia.in/index.php?title=Spot_Exchanges
Q Source: NSEL Scam

8. The Constitution (122nd Amendment) (GST) Bill, 2014 must be passed by at least half of all state assemblies in India
by
A. Two-third majority of the assembly present and voting.
B. Absolute majority of the total membership of the house.
C. Simple majority in form of a resolution.
D. Concurrence of the Speaker of the House and agreed to by the Governor of the State.
User Answer : C
Correct Answer : C
(C) Insights Active Learning. | All rights reserved.

www.insightsias.com

TEST - 1

Total Marks :

200

Mark Scored :

110.67

Answer Justification :
Learning: Amendments that seek to change federal provisions of the Constitution will be first passed in the two
houses separately by absolute and special majority.
They afterwards also need to be ratified by legislatures of at least half of the states by resolutions.
Only after this, the bill will be sent for presidential assent, and it becomes an Act.
Assam recently became the first state to ratify the GST Bill.
Q
Source:http://www.thehindu.com/news/national/other-states/assam-becomes-first-state-to-ratify-gst-bill/article898
0777.ece

9. With regard to pollution and climate change, why environment regulators are targeting Diesel cars more than Petrol
cars?
1. Diesel cars have higher emissions of NOx and particulate matter than petrol cars.
2. High sulphur content emitted by Diesel cars lead to acid rain.
Select the correct answer using the codes below.
A. 1 only
B. 2 only
C. Both 1 and 2
D. None
User Answer : B
Correct Answer : C
Answer Justification :
Concept: Naturally occurring crude can be burnt directly to produce energy, but because different hydrocarbons
have different boiling points, the combustion will be very uneven.
As such, crude oil is refined into compounds like kerosene, LPG, paraffin wax, naphtha, etc., apart from
petrol and diesel.
Longer the carbon chain, higher is the probability of finding other elements like sulphur and nitrogen
attached to it. Petrol is a more uniform carbon compound than diesel as petrol has shorter chains than
Diesel.
Crucially, diesel has higher sulphur content.
Justification: Statement 2:Emissions from diesel vehicles, especially particulate matter and black carbon, are the
major reasons for air pollution as well. Sulphates, like sulphur dioxide, released in the atmosphere may lead to acid
rain. So, statement 2 is correct.
Petrol exhaust produces carbon monoxide (CO) and some nitrous oxides (NOx) during combustion.
Statement 1:The CO combines with oxygen again to produce carbon dioxide (CO2), which is what comes out of
the exhaust pipes of petrol vehicles.
The relatively high nitrogen and sulphur content in diesel results in the formation of higher amounts of
NOx, and particulate matter containing mainly sulphurous compounds.
CO2 is a greenhouse gas, and contributes to global warming; NOx and particulate matter carry major health
risks.

(C) Insights Active Learning. | All rights reserved.

www.insightsias.com

TEST - 1

Total Marks :

200

Mark Scored :

110.67

Q Source: Improvisation: http://www.downtoearth.org.in/news/sc-allows-diesel-suvs-back-into-the-capital-55281

10. Which of the following is NOT vital for the establishment or success of a democracy?
A. Presence of Rule of Law.
B. Granting fundamental rights to the population.
C. Political equality of citizens to run for public offices.
D. Direct elections of all representatives.
User Answer : D
Correct Answer : D
Answer Justification :
Justification:Option A: Rule of law ensures that democracy doesnt turn into Rule of Men.
Option B: Fundamental rights guard the citizens from the tyranny and despotism of the rulers.
Option C: If a certain class of citizens are banned (without any intelligible criteria) from participating in elections
or running for public offices, a democracy may not be sustained.
Option D: Many democracies practice indirect election of representatives, for e.g. President of USA is indirectly
elected. Voters elect the Electoral College, which then elects the President.
In India, Rajya Sabha MPs are indirectly elected. So, direct election of all representatives is not crucial for a
democracy.
Q Source: Page 32: NCERT 6th: Social and Economic Life I

11. In India, the public health service is a chain of health centres and hospitals run by the government. Under it, Primary
Health Centres (PHCs) serve
A. Many villages in a rural area.
B. Specific wards in a Municipality.
C. Several hundreds of villages at the block level.
D. As attachments of the District Hospitals in sub-urban areas.
User Answer : A
Correct Answer : A
Answer Justification :
Learning: At the village level there are sub-health centres where there is usually a nurse and a village health
worker. They are trained in dealing with common illnesses.
They work under the supervision of doctors at the Primary Health Centre (PHC) which is next in the hierarchy.
Such a centre covers many villages in a rural area.
At the Block level, there is usually a Community Health Centre with more facilities than at PHC.
At the district level is the District Hospital that also supervises all the health centres.
Q Source: Page 24: NCERT 7th: Social and Economic Life II

12. Which of the following clearly shows the importance of carbon dioxide in earths atmosphere?
1. It is essential for the growth of plants.
2. It is the most abundant gas in the atmosphere after nitrogen and oxygen.
(C) Insights Active Learning. | All rights reserved.

www.insightsias.com

TEST - 1

Total Marks :

200

Mark Scored :

110.67

3. It absorbs the heat directly incoming from the Sun thus warming the planet.
4. Ocean absorbs carbon dioxide from the atmosphere to form carbonate rocks.
Select the correct answer using the codes below.
A. 1 and 4 only
B. 2 and 3 only
C. 1, 3 and 4 only
D. 1, 2, 3 and 4
User Answer : C
Correct Answer : A
Answer Justification :
Justification: Statement 1 : Green plants use CO2 gas to prepare their food through photosynthesis.
Statement 2: Argon is the next abundant gas.

Statement 3: Carbon dioxide in the atmosphere does not absorb the incoming visible radiations coming from the
sun blocks the out going infrared radiations radiated by earth. By absorbing infrared radiations, the atmosphere
gets heated. This is known as Green house Effect. The heated atmosphere keeps the earthworm. Thus carbon
dioxide helps in keeping the earth warmer by blocking the infrared radiations.
Statement 4: When carbon dioxide (CO2) is absorbed by seawater, chemical reactions occur that reduce seawater
pH, carbonate ion concentration, and saturation states of biologically important calcium carbonate minerals.
It is called "ocean acidification".
Calcium carbonate minerals are the building blocks for the skeletons and shells of many marine organisms.
Learning: Volcanic outgassing and wildfires are two significant natural sources of CO2 in Earth's atmosphere.
Respiration, the process by which organisms liberate energy from food, emits carbon dioxide.
Carbon dioxide is the most abundant gas in the atmospheres of Mars and Venus.
Q Source: Improvisation: Page 35: NCERT 6th Geography: The Earth Our Habitat

13. Consider the following about Unstructured Supplementary Service Data (USSD) that is being utilized by many
corporations in India for service delivery.
1. Use of USSD is limited due to its requirement of high speed internet.
2. It can be used for serving the banking needs of customers.
Which of the above is/are correct?
(C) Insights Active Learning. | All rights reserved.

www.insightsias.com

TEST - 1

Total Marks :

200

Mark Scored :

110.67

A. 1 only
B. 2 only
C. Both 1 and 2
D. None
User Answer :
Correct Answer : B
Answer Justification :
Justification: USSD is a GSM based technology i.e. used to send text between a mobile phone and an application
program in the network.
Unlike Short Message Service (SMS) messages, USSD messages create a real-time connection during a USSD
session. So they are more responsive than services that use SMS.
Statement 1: So, it can be used even without any internet connection as it does not require any data. 1 will be
wrong.
Statement 2: The Union Bank of India (UBI) has recently launched an Unstructured Supplementary Service Data
(USSD) based *99# mobile application for their customers for basic banking needs.
It was launched in partnership with National Payments Corporation of India (NPCI), the umbrella organisation for
all retail payments system in the country.
Q
Source:http://economictimes.indiatimes.com/industry/banking/finance/banking/-union-bank-of-india-launches-uss
d-based-mobile-app-for-basic-banking-needs-with-npci/articleshow/53673011.cms

14. The Archaeological Ruins of the location comprise the first great urban centre of the Indus civilization with burnt
brick structures. It is a UNESCO World Heritage Site and recently a movie on it was directed by Ashutosh Gowariker. It
is?
A. Burzahom
B. Inamgaon
C. Lothal
D. None of the above
User Answer : D
Correct Answer : D
Answer Justification :
Learning: It is Mohenjodaro. The Archaeological Ruins at Moenjodaro are the best preserved urban settlement in
South Asia dating back to the beginning of the 3rd millennium BC, and exercised a considerable influence on the
subsequent development of urbanization.
The archaeological ruins are located on the right bank of the Indus River
It was one of the largest settlements of the ancient Indus Valley Civilization.
The Archaeological Ruins at Moenjodaro are being protected by National and Regional laws including the
Antiquities Act 1975
The discovery of Moenjodaro in 1922 revealed evidence of the customs, art, religion and administrative
abilities of its inhabitants.

(C) Insights Active Learning. | All rights reserved.

www.insightsias.com

TEST - 1

Total Marks :

200

Mark Scored :

110.67

QSource: Recent movie Improvisation: Chapter 4: NCERT 6th History: Our Pasts-I

15. Which of the following forest types covers the largest area in India?
A. Tropical Moist Deciduous Forest.
B. Tropical Evergreen Forests.
C. Littoral and Swamp Forest.
D. Sub-Tropical Broad Leaved Hill Forest.
User Answer : A
Correct Answer : A
Answer Justification :
Justification: FAO has provided the relevant statistics, where largest Tropical moist deciduous is followed by
tropical dry deciduous. So, deciduous forests cover the largest forest area in India, more than 60%!
Tropical wet evergreen forests follow and then semi-evergreen forests.

(C) Insights Active Learning. | All rights reserved.

www.insightsias.com

TEST - 1

Total Marks :

200

Mark Scored :

110.67

Q Source: Improvisation: Page 59: NCERT 6th Geography: The Earth Our Habitat

16. In India, rocks of which of the following regions was formed the earliest?
A. Khasi hills
B. Lower Shiwaliks
C. Indo-Gangetic plain
D. Deccan Traps
User Answer : B
Correct Answer : D
Answer Justification :
Justification: Geoglogy of India can be categorized in following eras, in increasing chronological order.
Precambrian super-eon
Phanerozoic
Palaeozoic
Lower Paleozoic
Upper Paleozoic
Mesozoic
Cenozoic
Tertiary period
Quaternary period
Option (a): Khasi hills, one of youngest, belong to Tertiary period (to which a large area in North-east also
belongs).
Option (b): Shiwaliks are part of Himalayas, so given the reasoning below, this (b) will also be incorrect.
Option (c): The alluvium which is found in the Indo-Gangetic plain belongs to the Quaternary period. It was
eroded from the Himalayas by the rivers and the monsoons. Since Himalayas were formed after Deccan traps, and
are very young, (c) cant be the answer.

(C) Insights Active Learning. | All rights reserved.

www.insightsias.com

10

TEST - 1

Total Marks :

200

Mark Scored :

110.67

The Deccan plateau in India is one of the oldest plateaus in India made mainly from igneous rocks.
We will be covering more questions on this area in later tests.
Q Source: Improvisation: Page 42: NCERT 6th Geography: The Earth Our Habitat

17. Consider the following about the United Nations Commission on Sustainable Development (CSD).
1. It was established by the Conference of Parties (COP) to the UNFCCC.
2. Its primary duty is to enforce the binding agreements signed under the UNFCCC.
3. It meets every ten years to chart the course of sustainable development in both developed and developing
countries.
Select the correct answer using the codes below.
A. 1 only
B. 2 and 3 only
C. 3 only
D. None of the above
User Answer :
Correct Answer : D
Answer Justification :
Justification: Statement 1: It was established by the UN General Assembly in December 1992 to ensure effective
follow-up of United Nations Conference on Environment and Development (UNCED), also known as the Earth
Summit.
At the United Nations Conference on Sustainable Development (Rio 20), Member States agreed to establish a high
level political forum that replaced the Commission on Sustainable Development. The body no longer exists.
Statement 2: The Commission was responsible for reviewing progress in the implementation of Agenda 21 and
the Rio Declaration on Environment and Development; as well as providing policy guidance to follow up the
Johannesburg Plan of Implementation (JPOI) at the local, national, regional and international levels.
Statement 3: The CSD met annually in New York, in two-year cycles, opening its sessions to broad participation
from both governmental and non-governmental actors.
Q Source: https://sustainabledevelopment.un.org/intergovernmental/csd

18. Non-performing Assets (NPAs) are loans made by a bank or finance company on which repayments or interest
payments are not being made on time. How do high NPAs affect the citizens (customers) in India?
1. They may lead to an increase in the cost of capital for borrowers.
2. They may lead to bank defaults and thus lower peoples confidence in the banking system affecting their savings
ratio.
Which of the above is/are correct?
A. 1 only
B. 2 only
C. Both 1 and 2
D. None
User Answer : C
Correct Answer : C
Answer Justification :
Justification: Statement 1: It results in inflating the cost of capital for economic activities and banks may charge
(C) Insights Active Learning. | All rights reserved.

www.insightsias.com

11

TEST - 1

Total Marks :

200

Mark Scored :

110.67

higher interest rates on some products to compensate NPAs.


NPA is any asset of a bank which is not producing any income. It affects the profitability & liquidity of the banks.
Statement 2: It adversely affects the value of bank in terms of market credit and widens assets and liability
mismatch. This may even lead to bank defaults.
It shakes consumers confidence in the banking deposits, and forces them to withdraw savings in banks, thus
lowering the savings ratio.
Learning: Savings ratio is the total amount of a persons income not spent on consumption and invested in either
bank deposits, bonds or other market instruments.
In any economy, savings finance investments. Higher savings increase liquidity in the market and reduce the cost
of credit resulting in higher economic growth.
So, it is very important that the banking system of an economy is robust.
Q Source:http://www.thehindubusinessline.com/opinion/npas-banks-alone-are-not-to-blame/article8955877.ece

19. If we did NOT use an Indian Standard Time, the actual time difference between which of the following places
would be more than 1 hour?
A. Patna and Kolkata
B. Dwarka and Dibrugarh
C. Chandigarh and Rewari
D. Sultanpur and Kanpur
User Answer : B
Correct Answer : B
Answer Justification :
Justification:Sultanpur and Kanpur both are in UP, Chandigarh and Rewari both in Haryana; Patna and Kolkata
situated not far by.
The difference in time is created by longitude differences. Difference of 15 degrees creates a difference of 1 hour.
Based on the longitudes of Dwarka (Gujarat) and Dibrugarh (Assam), there will bea difference of about 1 hour and
45 minutes in thelocal times of Dwarka and Dibrugarh.
Q Source: Page 15: NCERT 6th Geography: The Earth Our Habitat

20. Which of the following correctly differentiates the events of equinox with that of solstice?
1. In a solstice only a particular tropic receives direct vertical rays of the Sun unlike in an equinox the equator
receives direct vertical rays of Sun.
2. Solstice is caused only due to rotation of the earth, whereas equinox is caused only due to revolution of the earth.
Which of the above is/are correct?
A. 1 only
B. 2 only
C. Both 1 and 2
D. None
User Answer : A
Correct Answer : A
Answer Justification :
(C) Insights Active Learning. | All rights reserved.

www.insightsias.com

12

TEST - 1

Total Marks :

200

Mark Scored :

110.67

Justification: Statement 1: The image below explains statement 1. Notice how on Summer equinox, Tropic of
Cancer receives direct overhead rays of the Sun, while in Winter solstice, it is the Tropic of Capricorn.
On an equinox, when days and nights are equal in duration, Sun is directly overhead the equator, causing equal
days and nights.

Statement 2:Both are caused due to axial tilt of the earth, its revolution and also rotation (to cause day and night).
The diagram above makes it clear.
Q Source: Page 19: NCERT 6th Geography: The Earth Our Habitat

21. Consider the following about the Panchayat Secretary.


1. She is elected by the Gram Panchayat from amongst its members.
2. She is responsible for calling the meeting of the Gram Sabha and Gram Panchayat and keeping a record of the
proceedings.
Which of the above is/are correct?
A. 1 only
B. 2 only
C. Both 1 and 2
D. None
User Answer : B
Correct Answer : B
Answer Justification :
Justification: Statement 1: She is not an elected person but is appointed by the government.
Statement 2: Role of Panchayat Secretary is defined by State rules/regulations and laws. In the state of AP, some
of her roles are:
She shall maintain the required registers of the Gram Panchayat.
She is subordinate to the Gram Panchayat and shall function under the control of the Sarpanch of the Gram
Panchayat.
She shall attend the meeting of the Gram Panchayat or any committee thereon.
She shall take the responsibility of the protection of Panchayat lands and properties.
She shall report of encroachments, damage or misuse of Panchayat lands and buildings if any to the higher
authorities.
She shall assist Grama Sabha in the identification of beneficiaries, disbursement of loans and their
recoveries.
(C) Insights Active Learning. | All rights reserved.

www.insightsias.com

13

TEST - 1

Total Marks :

200

Mark Scored :

110.67

Q Source: Page 44: NCERT 6th: Social and Economic Life I

22. The Arctic Ocean is located within the Arctic Circle and surrounds the North Pole. It is connected with the Pacific
Ocean by a narrow stretch of shallow water known as
A. Berring strait
B. Alaska Strait
C. Isthmus of Faribanks
D. Norton Sound
User Answer : A
Correct Answer : A
Answer Justification :
Justification: It connects the Pacific and Arctic oceans.
The International Date Line runs around the Bering strait.
This strait separates the United States and Russia by around 85 km, with a water depth that measures only 3050
meters!
In the last few decades some factions have discussed the construction of a bridge over the strait, however, financial
and weather concerns have continually stalled the project.

(C) Insights Active Learning. | All rights reserved.

www.insightsias.com

14

TEST - 1

Total Marks :

200

Mark Scored :

110.67

Q Source: Page 35: NCERT 6th Geography: The Earth Our Habitat

23. Communicable diseases account for a large burden of diseases in India. Which of these may NOT help controlling
their spread?
A. Improving sanitation and drinking water in the locality
B. Isolating infected persons
C. Reducing intake of salt in food
D. Using only new syringes for blood transfusion and disposing old used ones
User Answer : C
Correct Answer : C
Answer Justification :
Learning: Communicable diseases are caused by pathogens passed from one human to another.
Pathogens are viral, bacterial, parasitic and fungal. Methods of transmission include mucus, blood, breath, saliva
and sexual contact.
Contaminated surfaces, such as doorknobs, counter tops and playground equipment, provide a medium for passing
disease from one human to another.
So, all options other than C are correct.
Option C can control cardiovascular non-communicable diseases.
th

Q Source: Page 29: NCERT 7 : Social and Economic Life II

(C) Insights Active Learning. | All rights reserved.

www.insightsias.com

15

TEST - 1

Total Marks :

200

Mark Scored :

110.67

24. Which of the following factors contribute to the evolution of landforms on earth?
1.
2.
3.
4.

Movement of magma within the earth


Growth and decay of vegetation
Erosion and deposition
Frost action
Select the correct answer using the codes below.
A. 1 and 3 only
B. 2 and 4 only
C. 1, 3 and 4 only
D. 1, 2, 3 and 4
User Answer : C
Correct Answer : D
Answer Justification :
Justification: Statement 1: It causes plate movements (tectonics), results in volcanic eruptions and is therefore a
significant factor in the evolution of landforms on earth.
Statement 2:If Vegetation cover is high, it protects the surface from rain splash as root mass is sufficient to
stabilize the materials on the slope.
Even surface runoff become less effective in carving out landforms in areas where there is dense vegetation, since
there is little scope for soil erosion.
Statement 3:Self-explanatory. For e.g. all the plains that you are the result of active erosion and deposition by
water.
Statement 4:When water freezes to ice, its volume increases. Under specific circumstances, this expansion is able
to displace or fracture rocks where water exist in its pores. Repeated frost action thus weathers (breaks) the rocks.
In some mountains, there are permanently frozen rivers of ice. They are called glaciers. Glaciers move at a
very slow rate. When they do, they erode the soil beneath them.
Also, formation of glaciers and their retreat affects the soil profile of the region and thus the landforms.
Q Source: Page 39-40: NCERT 6th Geography: The Earth Our Habitat

25. In India, women got universal suffrage rights with the


A. 61st Constitutional Amendment, 1989
B. Enactment of Government of India Act 1935
C. Commencement of the Indian Constitution
D. Act of 1858 passed by the British Parliament
User Answer : C
Correct Answer : C
Answer Justification :
Justification:Option A: The 61st Amendment Act lowered the voting age from 21 to 18 years thus introducing
adult suffrage. But, women already enjoyed the right to vote in India.
Option C: The constitution provided for universal franchise (21 years age or later) without discrimination of caste,
creed or gender.
Some British Acts did increase the level of electoral participation in India, but they never introduced universal
adult franchise. We will be covering such Acts in the coming tests.
(C) Insights Active Learning. | All rights reserved.

www.insightsias.com

16

TEST - 1

Total Marks :

200

Mark Scored :

110.67

Q Source: Page 32: NCERT 6th: Social and Economic Life I

26. Northern blot is a laboratory method used to


A. Determine the hereditary lineages in forensic investigation
B. Examine changes in plant physiology
C. Copy the information from one strand of genome to the other
D. Detect specific RNA molecules among a mixture of RNA
User Answer :
Correct Answer : D
Answer Justification :
Justification: Northern blotting can be used to analyze a sample of RNA from a particular tissue or cell type in
order to measure the RNA expression of particular genes.
Application: Northern blotting allows one to observe a particular gene's expression pattern between tissues,
organs, developmental stages, environmental stress levels, pathogen infection, and over the course of treatment.
Learning: Some other related terms are:
A microarray is a laboratory tool used to detect the expression of thousands of genes at the same time.
A proteome is the complete set of proteins expressed by an organism.
Q Source: Nature.com http://www.news-medical.net/life-sciences/Northern-Blot-RNA-Blot.aspx Related to
UPSC 2016 Prelims question on Transcriptome

27. The State of Madhya Pradesh lying in Central India shares land boundary with which of the following states?
1.
2.
3.
4.
5.

Bihar
Jharkhand
Uttar Pradesh
Telangana
Odisha
Select the correct answer using the codes below.
A. 1, 2 and 3 only
B. 3 only
C. 3, 4 and 5 only
D. 2 and 4 only
User Answer : B
Correct Answer : B
Answer Justification :
Justification: Both Bihar and Jharkhand dont touch MP, as UP land boundary comes before that.
Extent of Maharashtra should be noted carefully. Many map based questions use the extent of MH and UP to
confuse you in state land borders and their extent.

(C) Insights Active Learning. | All rights reserved.

www.insightsias.com

17

TEST - 1

Total Marks :

200

Mark Scored :

110.67

Q Source:Map based questions: India

28. Which of the following statements about the Financial Action Task Force (FATF), established in 1989 by G-7, is
INCORRECT?
A. It is an intergovernmental policy making body.
B. Its mandate covers setting international standards for combating money laundering and terrorist financing.
C. It does not deal with proliferation of weapons of mass destruction which are dealt by the UNSC and multilateral export control regimes.
D. It works to identify national-level vulnerabilities with the aim of protecting the international financial
system from misuse.
User Answer : A
Correct Answer : C
Answer Justification :
Justification:Option B: The objectives of the FATF are to set standards and promote effective implementation of
legal, regulatory and operational measures for combating money laundering, terrorist financing and other related
threats to the integrity of the international financial system.
The FATF is therefore a policy-making body which works to generate the necessary political will to bring
about national legislative and regulatory reforms in these areas.
Option C: The FATF has developed a series of Recommendations that are recognised as the international standard
for combating of money laundering and the financing of terrorism and proliferation of weapons of mass
destruction
Option D: The FATF monitors the progress of its members in implementing necessary measures, reviews money
laundering and terrorist financing techniques and counter-measures, and promotes the adoption and
implementation of appropriate measures globally.
In collaboration with other international stakeholders, the FATF works to identify national-level
vulnerabilities with the aim of protecting the international financial system from misuse.
The FATF's decision making body, the FATF Plenary, meets three times per year.
Q
Source:http://www.thehindu.com/news/national/india-seeks-us-help-to-crack-pakistan-terror-fund-trail/article896
1156.ece
(C) Insights Active Learning. | All rights reserved.

www.insightsias.com

18

TEST - 1

Total Marks :

200

Mark Scored :

110.67

29. Which of the following forest types is common to Andaman and Nicobar Islands, parts of North-Eastern states and a
narrow strip of the Western slope of the Western Ghats?
A. Mangrove forests
B. Temperate forests
C. Tropical moist forests
D. Coral forests
User Answer : C
Correct Answer : C
Answer Justification :
Justification:Option (a): Mangroves are found in A&N islands and even near the western coast of India, but not in
NE states, or on the Western slope of Western Ghats.
Option (b): Temperate forests are found in temperate latitudes or at higher altitudes. A&N islands do not host
temperate forests.
Option (d): Corals are found largely in A&N, Lakshadweep, Gulf of Mannar (near TN) and Gulf of Kutch.
Option (c): Tropical moist forests include evergreen forests; Tropical Semi-Evergreen Forests, Tropical Moist
Deciduous Forests and Littoral and Swamp Forests.
The Middle Andamans harbours mostly moist deciduous forests. North Andamans is characterised by the
wet evergreen type.
Western Ghats and NE states harbour both deciduous and evergreen forests.
We will be covering more questions specific to vegetation in various parts of India in later tests.
Q Source: Page 58: NCERT 6th Geography: The Earth Our Habitat

30. Jupiter, Saturn and Uranus have rings around them. These rings consist of
A. Satellites of these planets only.
B. Icy dust, large rocks and moons of the planets.
C. Comets and asteroids only.
D. Magnetospheric flux of the planet.
User Answer : B
Correct Answer : B
Answer Justification :
Learning: The composition of ring particles varies; they may be silicate or icy dust. Larger rocks and boulders
may also be present. Sometimes rings will have small moons that orbit near the inner or outer edges of rings or
within gaps in the rings.\
For e.g. the ring swirling around Saturn consists of chunks of ice and dust.
Recent evidence suggests that ring systems may be found around other types of astronomical objects, including
minor planets, moons, and brown dwarfs.
Q Source: Page 2 (Side Box): NCERT 6th Geography: The Earth Our Habitat

31. Consider the following species and the regions where they are found in. Species Region A. Wild Ass 1. Assam B.
Wild Goat 2. Runn of Kutch C. One-horned Rhinoceros 3. Himalayan region
The correct match for the above will be?
(C) Insights Active Learning. | All rights reserved.

www.insightsias.com

19

TEST - 1

Total Marks :

200

Mark Scored :

110.67

A. A-1, B-3, C-2


B. A-2, B-3, C-1
C. A-3, B-1, C-2
D. A-2, B-1, C-3
User Answer : B
Correct Answer : B
Answer Justification :
Justification:Elephants and one-hornedrhinoceroses (categorized vulnerable as per IUCN) are found in the forests
of Assam, especially Kaziranga national park.
Camels andwild asses are found in the Great Indian desertand the Rann of Kuchchh, which is also the largest
wildlife sanctuary in India. As of 2016, the Indian wild ass is listed as Near Threatened by IUCN.
Wild goats, snow leopards,bears, etc. are found in the Himalayan region. Some of these wild goats are reared for
the famous Pashmina wool.
Q Source: Page 63: NCERT 6th Geography: The Earth Our Habitat

32. The Future We Want document was a result of which of the following conferences?
A. Earth Summit 2012 at Rio.
B. Warsaw Climate Change Agreement.
C. Sustainable Development Goals (SDGs) meeting at UN General Assembly.
D. The First UN conference on Sustainable development .
User Answer :
Correct Answer : A
Answer Justification :
Learning: It was a nonbinding document, "The Future We Want," a work paper.
In it, the heads of state of the 192 governments in attendance renewed their political commitment to sustainable
development and declared their commitment to the promotion of a sustainable future.
The document largely reaffirms previous action plans like Agenda 21.
Q Source: https://sustainabledevelopment.un.org/rio20/futurewewant

33. Consider the following statements about the historical inscriptions in India.
1. All the edicts of Asoka engraved in the Kharoshthi and Brahmi scripts are in the Prakrit language.
2. With the rise of the Guptas, Prakrit became the predominant language of Indian epigraphs.
Which of the above is/are correct?
A. 1 only
B. 2 only
C. Both 1 and 2
D. None
User Answer : A
Correct Answer : A
Answer Justification :
Justification: Statement 1: This is with reference to Epigraphical Studies in India - Sanskrit and Dravidian
Language of the inscriptions.
(C) Insights Active Learning. | All rights reserved.

www.insightsias.com

20

TEST - 1

Total Marks :

200

Mark Scored :

110.67

We are quoting verbatim from ASIs website, The language of the earliest written records, viz., the
Asokan edicts, is Prakrit. Besides Prakrit, Asokan edicts are written in Greek and Aramaic languages also.
All the edicts of Asoka engraved in the Kharoshthi and Brahmi scripts are in the Prakrit language.
Thus, originally the language employed in the inscriptions was Prakrit and Sanskrit was adopted in the
inscriptions only at a later period.
Statement 2:After the period of Asoka, the use of the Prakrit language continued in inscriptions for a few more
centuries.
In north India, Prakrit was replaced by Sanskrit about the end of 3rd century A.D. while this change took
place about a century later in south India.
From the 4th century onwards, with the rise of the Guptas, Sanskrit became the predominant language of
Indian epigraphs. So, 2 is wrong.
This is because Guptas were great patrons of Brahmanism and Sanskrit language.
Learning: The contemporary rulers of the Guptas, in central India and parts of the Deccan, like the Vakatakas, the
Kadambas and later Gangas of Karnataka and the Pallavas in south India also employed Sanskrit in their
inscriptions.
The copper-plate charters of the Pallavas, the Cholas and the Pandyas are written in both Sanskrit and Tamil
languages.
Q Source:Improvisation: Page 5: NCERT 6thHistory: Our Pasts-I

34. Consider the following about the Rock Shelters of Bhimbetka.


1.
2.
3.
4.

Bhimbetka derives its name from the Atharva Veda.


Mesolithic as well as Medieval period rock paintings can be found in the caves.
It is a UNESCO World Heritage Site.
Bhimbetka paintings are highly secular in nature.
Select the correct answer using the codes below.
A. 2 and 4 only
B. 1 and 3 only
C. 2 and 3 only
D. 1 and 4 only
User Answer : C
Correct Answer : C
Answer Justification :
Justification: Statement 1: Bhimbetka owes its name to the characters of the longest epic in the world, the
Mahabharata. It is believed that when the five brothers, called Pandavas, were banished from their kingdom, they
came here and stayed in these caves, the massive rocks seating the gigantic frame of Bhima, the second Pandava.
Statement 2:Bhimbetka is an archaeological treasure.The rock paintings have numerous layers belonging to
various epochs of time, ranging from the Upper Palaeolithic, Mesolithic Age to the protohistoric, early historic and
medieval periods. The most ancient scenes here believed to be commonly belonging to the Mesolithic Age. So, 2
is correct.
Statement 3:You can read more about it here http://asi.nic.in/asi_monu_whs_rockart_bhimbetka_detail.asp
Statement 4:Animals such as bison, tiger, rhinoceros, wild boar, elephants, monkeys, antelopes, lizards, peacocks
etc. have been abundantly depicted in the rock shelters. Popular religious and ritual symbols also occur frequently.
So, 4 is false.

(C) Insights Active Learning. | All rights reserved.

www.insightsias.com

21

TEST - 1

Total Marks :

200

Mark Scored :

110.67

Q Source: Improvisation: Page 14-15: NCERT 6th History: Our Pasts-I

35. Mikir hills, Mishmi hills and Barail range are located in
A. North-eastern India
B. Southern India
C. Northern Ladakh
D. Western Aravallis
User Answer : A
Correct Answer : A
Answer Justification :
Learning: These hills are famous for their tribal inhabitants. We will be covering specific tribes of these regions
in later tests.
Note the order of the hills and the state they are located in.

Q Source: Map-based questions : India

36. Consider the following statements about the India Nuclear Insurance Pool (INIP).
1. It was launched as per the mandatory provision under the Civil Liability for Nuclear Damage Act (CLND), 2010.
2. The pool provides capacity for insurance coverage to citizens affected by nuclear hazards in India.
3. It will fund future development of those nuclear reactors which involve considerable risks due to geology or
radiation.
Select the correct answer using the codes below.
A. 1 only
B. 2 and 3 only
C. 1 and 2 only
D. 1, 2 and 3
User Answer : C
Correct Answer : A
Answer Justification :
Justification: Statement 1: Based on Obamas visit, a INIP was negotiated, which then came into effect via the
CLND Act.
(C) Insights Active Learning. | All rights reserved.

www.insightsias.com

22

TEST - 1

Total Marks :

200

Mark Scored :

110.67

Statement 2: The pool provides capacity for insurance coverage to operators and suppliers for any nuclear liability
towards third party. INIP also will offer policies on the nuclear operators liability insurance policy and a nuclear
suppliers special contingency (against right to recourse) insurance policy.
Statement 3: There is no such provision. It is an insurance pool only, not a development pool.
Q Source: http://www.thehindu.com/business/nuclear-suppliers-insurance-policy-unveiled/article8981843.ece

37. Consider the following about the S.A.M.O.A. Pathway.


1. It is a part of the Paris Climate Change Agreement 2015.
2. It focuses on those communities in developing countries that derive their livelihood exclusively from forestry and
use of other natural resources.
Which of the above is/are correct?
A. 1 only
B. 2 only
C. Both 1 and 2
D. None
User Answer :
Correct Answer : D
Answer Justification :
Justification: SAMOA was an outcome document of the Third International Conference on Small Island
Developing States (SIDS), the 'SIDS Accelerated Modalities of Action (SAMOA) Pathway.
The Samoa Pathway recognizes the adverse impacts of climate change and sea-level rise on SIDS efforts to
achieve sustainable development as well as to their survival and viability, and addresses economic development,
food security, disaster risk reduction (DRR) and ocean management, among other issues.
Since the agreement came about in 2014, it cannot be a part of the Paris Climate agreement 2015. So, 1 is wrong.
Q Source: Last 2 years current affairs: https://sustainabledevelopment.un.org/sids2014/samoapathway

38. Consider the following. Communities seen in news Belong to 1. Tuaregs A. Sahara Desert 2. Sinhalas B. Bhutan 3.
Rakhine C. Sri Lanka
Which of these is/are correctly matched?
A. 2 and 3
B. 1 only
C. 1 and 3 only
D. 1 and 2 only
User Answer : B
Correct Answer : B
Answer Justification :
Justification: Statement 1: These are people with a nomadic pastoralist lifestyle. They are the principal
inhabitants of the vast Sahara Desert.
They raged the Tuareg Rebellion (2012) which was an early stage of the Northern Mali conflict waged against the
Malian government. Their goal was attaining independence for the northern region of Mali, known as Azawad.
Statement 2: Sinhalas are the major community of Sri Lanka often in conflict with the Tamils of SL.
Statement 3: Rakhine is a state in Myanmar, and also a principal ethnic community. The 2012 Rakhine State riots
(C) Insights Active Learning. | All rights reserved.

www.insightsias.com

23

TEST - 1

Total Marks :

200

Mark Scored :

110.67

were a series of conflicts primarily between ethnic Rakhine Buddhists and Rohingya Muslims in northern Rakhine
State.
Q Source: Often in news Improvisation: Page 73: Geography NCERT 7th: Our Environment

39. Consider the following about the Malware Infection Index 2016 (MII 2016) for Asia-Pacific region.
1. It was released by Internet Corporation for Assigned Names and Numbers (ICANN).
2. The Index pin points India to be the most severely affected country in the entire region.
Which of the above is/are correct?
A. 1 only
B. 2 only
C. Both 1 and 2
D. None
User Answer :
Correct Answer : D
Answer Justification :
Justification: Statement 1: Microsoft Asia announced the launch of its Malware Infection Index 2016 (MII2016),
which identifies the key malware threats in the region and ranks markets in Asia Pacific according to how much
they are affected.
The region is especially vulnerable with emerging markets most at risk of malware threats.
Statement 2: The image below clearly shows that countries like Pakistan and some ASEAN nations are most
affected. India is less severely affected.

(C) Insights Active Learning. | All rights reserved.

www.insightsias.com

24

TEST - 1

Total Marks :

200

Mark Scored :

110.67

Q
Source:http://timesofindia.indiatimes.com/tech/tech-news/Pakistan-tops-the-malware-index-list-India-8th-Micros
oft/articleshow/52636322.cms

40. Einstein Ring can be observed when


A. All planets in the solar system fall in a common line
B. A pair of distant galaxies from earth aligns in a special position with respect to earth
C. Solar eclipse is just about to end
D. Volcanic eruptions occur at the craters of the Jovian planets
User Answer : C
Correct Answer : B
Answer Justification :
Learning: Einstein Ring was discovered recently by an Institute of Chile.
"Einstein Ring", first predicted by Einstein's theory of General Relativity, is a rarely observed, astronomical
phenomenon that occurs when two distant galaxies are perfectly aligned, millions of light years apart.
Because the two galaxies are aligned so perfectly, light from the most distant, or source galaxy, is being
bent by the gravity of the closer one.
This makes the light from the furthest galaxy appear as an almost perfect circle when viewed from Earth.
(C) Insights Active Learning. | All rights reserved.

www.insightsias.com

25

TEST - 1

Total Marks :

200

Mark Scored :

110.67

An Einstein ring is a distorted image of a very distant galaxy, which is termed 'the source'.
Q Source:http://www.space.com/33095-nearly-perfect-einstein-ring-discovered-image.html

41. This ruler convened a Buddhist Council at Kashmir which eventually led to the recognition of the Mahayana sect of
Buddhism. He is?
A. Ashoka
B. Yavana
C. Kanishka
D. Kumaragupta
User Answer : C
Correct Answer : C
Answer Justification :
Justification & Learning: The Fourth Buddhist Council was held at Kundalvana, Kashmir in 72 AD under the
patronage of Kushan king Kanishka.
The president of this council was Vasumitra.
This council distinctly divided the Buddhism into 2 sects Mahayanaand Hinayana.
Another Fourth Buddhist Council was held at Tambapanni (one name of Sri Lanka) at Aloka Lena under
the patronage of Vattagamani-Abaya. This was Theravada Buddhist council.
Q Source: Page 103: NCERT 6th History: Our Pasts-I

42. Consider the following statements with reference to some of the officials in the History of India.
1. Nagara-shreshthi was the Chief Craftsman of the royal court.
2. Sarthavaha was the leader of bonded labourers.
3. Sandhi-vigrahika was a minister of war and peace.
Select the correct answer using the codes below.
A. 1 and 2 only
B. 2 and 3 only
C. 3 only
D. None of the above
User Answer : C
Correct Answer : C
Answer Justification :
Justification& Learning: Kings adopted a number of steps to win the support of men who were powerful,
eithereconomically, or socially, or because of theirpolitical and military strength.
Statement 1 and 2:Important men probably had a say in local administration. These included the nagarashreshthi or
chief banker or merchant of the city,the sarthavaha or leader of the merchant caravans,the prathama-kulika or the
chief craftsman, and thehead of the kayasthas or scribes.
Statement 3:Sometimes, one person held many offices. Forinstance, besides being a maha-dandanayaka,Harishena was a kumar-amatya, meaning an important minister, and a sandhi-vigrahika,meaning a minister
of war and peace.
Q Source: Page 116: NCERT 6th History: Our Pasts-I

43. Consider the following about National Mineral Exploration Trust (MET).
(C) Insights Active Learning. | All rights reserved.

www.insightsias.com

26

TEST - 1

Total Marks :

200

Mark Scored :

110.67

1. It grants mining leases for major minerals to after obtaining sanction from the State government.
2. It carries out detailed exploration for minerals using the funds accrued to it.
3. It invests in basic infrastructural development and implements welfare schemes in mining affected belts.
Select the correct answer using the codes below.
A. 1 only
B. 2 only
C. 1 and 3 only
D. 1 and 2 only
User Answer : C
Correct Answer : B
Answer Justification :
Justification: Statement 1: Major minerals are administered by the Central government. Moreover, this body
doesnt give sanction for mining leases. So, 1 is wrong.
Statement 2: The Mines and Minerals (Development & Regulation) Amendment Act, 2015, (MMRDA) mandated
the setting up of Mineral Exploration Trust.
It is a non-profit body by the Central Government for the purposes of regional and detailed exploration of minerals
using the funds accrued to it and in such manner as prescribed by the Central Government.
Statement 3: It is done by District Mineral Foundation. So, 3 is wrong.
Q Source: Other details can be seen here
http://www.business-standard.com/article/pti-stories/govt-notifies-norms-for-national-mineral-exploration-trust-11
5082100414_1.html

44. Atmosphere is held around the earth due to


A. Continuous magnetic activity in the Earths core.
B. Corona discharge from the Sun that reaches the earth consistently.
C. Gravitational pull of the earth.
D. Presence of clouds in the troposphere and other layers above it.
User Answer : C
Correct Answer : C
Answer Justification :
Justification: Celestial bodies like Moon dont have an atmosphere because their gravity isnt enough to hold the
atmosphere together with the body.
Since gravity is low, and gases are lighter, they escape from the celestial body.
This is not the case with earth, where gravitation is high enough to hold the gases together. Escape velocity of
gases on earth is much higher than that on some other celestial bodies like Moon.
Learning: Atmosphere also protects usfrom the harmful rays and scorching heat of the sun.It consists of a number
of gases, dust and water vapour.
The changes in the atmosphere producechanges inthe weather and climate.
Q Source: Page 3: Geography NCERT 7th: Our Environment

45. Ecosystem is formed, most appropriately, by the interaction of


(C) Insights Active Learning. | All rights reserved.

www.insightsias.com

27

TEST - 1

Total Marks :

200

Mark Scored :

110.67

A. Both biotic and abiotic factors of the environment.


B. All Living organisms present in the system.
C. Diverse Landforms found within a large zone.
D. Communities found near edge of a biome.
User Answer : A
Correct Answer : A
Answer Justification :
Justification: An ecosystem is asystem formed by theinteraction of allliving organisms (biotic) with each other
and with the physical andchemical factors (abiotic) ofthe environment inwhich they live, alllinked by transfer
ofenergy and material.
Biome is a larger region than ecosystem, and comprises many such ecosystems. So, D is incorrect.
We will be covering more questions on the hierarchies of ecology.
Q Source: Page 5: Geography NCERT 7th: Our Environment

46. Consider the following about state administration.


1. Governor of the State is appointed by the Central Government but the State Assembly may disapprove her
appointment in effect repealing it.
2. Governor of the state appoints the chief minister and other ministers only after obtaining the advice of the Central
Council of Ministers.
Which of the above is/are correct?
A. 1 only
B. 2 only
C. Both 1 and 2
D. None
User Answer : D
Correct Answer : D
Answer Justification :
Justification: Statement 1: Governor is appointed by the President. State has no say in either the appointment or
removal of the Governor. So, 1 is wrong.
Statement 2: Governor doesnt consult the Union government in making state appointments, even though the
Governor is an agent of the Centre.
She is bound to choose the leader of the largest party/coalition as the CM and others recommended by the CM as
ministers.
Q Source: Improvisation: Page 33: NCERT 7th: Social and Economic Life II

47. How the National Highways Grid proposed by the NHAI will be useful?
1. The Grid will improve connectivity between coastal areas and state capitals thus facilitating cargo movement.
2. It will help the government re-designate the National Highways for easy identification.
Which of the above is/are correct?
A. 1 only
B. 2 only
(C) Insights Active Learning. | All rights reserved.

www.insightsias.com

28

TEST - 1

Total Marks :

200

Mark Scored :

110.67

C. Both 1 and 2
D. None
User Answer : D
Correct Answer : C
Answer Justification :
Justification: Statement 1: The grid will connect 12 major ports, 26 state capitals and more than 45 cities and thus
help in quick evacuation and transport of cargo from one end to the other.
Statement 2: In India the lack of scientific road network pattern has created a problem for drivers who can't take a
straight road to reach from one place to the other.
National Highway Authority of India is proposing a National Highway Grid which will include 27
horizontal and vertical highway corridors spread across the country.
The corridors, spaced at a distance of 250 kilometres, will crisscross and connect with each other.
The preparation of the grids will help the government re-designate the NHs for easy identification. For
example, all even number NH grids can be identified for roads connecting east to west and odd number for
corridors joining north end to south
Q
Source:http://economictimes.indiatimes.com/news/economy/infrastructure/government-plans-national-highway-gr
ids-for-smooth-travel-27-corridors-identified/articleshow/52723683.cms

48. Greenhouse gases that are present in the atmosphere due to NATURAL activities is/are?
1.
2.
3.
4.
5.
6.

Chlorofluorocarbons
Methane
Nitrous oxide
Sulfur hexafluoride
Oxygen
Water vapour
Select the correct answer using the codes below.
A. 2, 4 and 5 only
B. 2, 3 and 6 only
C. 1, 3, 4, 5 and 6 only
D. 1, 2 and 6 only
User Answer : B
Correct Answer : B
Answer Justification :
Justification: Statement 1: CFC is a manmade gas, and so all options containing CFC will be wrong.
Statement 2:Methane is emitted by rice field, animals (digestive tracts), coal mines etc.
Statement 3:Nitrous oxide (N2O) is produced by fertilizer use, animal waste management, fossil fuel combustion,
industrial activities, soils under natural vegetation and the oceans. So, it is produced by both natural and human
activities. 3 will still be correct.
Statement 4:SF6 is an inorganicextremely potent greenhouse gas, which is an excellent electrical insulator. It is a
manmade gas.
Statement 5: Oxygen (O2) is not a GHG, ozone (O3) is.

(C) Insights Active Learning. | All rights reserved.

www.insightsias.com

29

TEST - 1

Total Marks :

200

Mark Scored :

110.67

Statement 6: Water vapour is a very important GHG, and it is obviously natural.


Q Source: Improvisation: Page 20: Geography NCERT 7th: Our Environment

49. Optically-Stimulated Luminescence is a dating technique that is used to find out


A. The last time quartz or feldspar sediment was exposed to light
B. Radiation emitted by antecedent water bodies
C. Age of tree barks by exposing them to ionizing radiation
D. Duration of photons incident on any surface
User Answer :
Correct Answer : A
Answer Justification :
Learning: Indian researchers have used dating techniques on animal remains and pottery fragments to conclude
that the Indus Valley settlements could be older than previously believed.
They also used the technique called Optically-Stimulated Luminescence.
It is a late Quaternary dating technique used to date the last time quartz sediment was exposed to light.
As sediment is transported by wind, water, or ice, it is exposed to sunlight and zeroed of any previous
luminescence signal.
Once this sediment is deposited and subsequently buried, it is removed from light and is exposed to low
levels of natural radiation in the surrounding sediment.
The technique measures their age using ionizing radiation.
You can read more here
http://timesofindia.indiatimes.com/india/Indus-era-8000-years-old-not-5500-ended-because-of-weaker-monsoon/a
rticleshow/52485332.cms
Q Source: As given above

50. Temperature in cities is much higher than that of villages due to


1.
2.
3.
4.
5.

Lower forest coverage in cities as compared to villages


Concrete and asphalt structure in cities capture and release heat
Vehicular emissions in cities release Greenhouse gases that trap heat
Cities are more densely populated as compared to villages
The structure of the economic activity of cities
Select the correct answer using the codes below.
A. 3, 4 and 5 only
B. 1, 2 and 3 only
C. 2, 4 and 5 only
D. 1, 2, 3, 4 and 5
User Answer : B
Correct Answer : D
Answer Justification :
Justification: Statement 1: Forest moderate temperature, cool the surroundings by transpiration and capture the
carbondioxide that traps heat. Deforestation and low forest cover contributes to urban heat islands.
Statement 2:The concrete andmetals in buildings and the asaphalt of roads get heated up during the day. This heat
is released during the night.

(C) Insights Active Learning. | All rights reserved.

www.insightsias.com

30

TEST - 1

Total Marks :

200

Mark Scored :

110.67

Also, the crowded high rise buildings of the citiestrap the warm air and thus raise the temperature ofthe cities.
Statement 3:Vehicle exhaust emits CO2, NOX and other GHGs that capture heat.
Statement 4 and 5:Density of population and industrial economic activity automatically increase the heat generated
per unit area.
Q Source:Improvisation: Page 24: Geography NCERT 7th: Our Environment

51. Direct Heat of the Sun can be felt immensely at Ladakh because of
A. High water vapour saturation in air
B. Day temperatures are very high in Ladakh
C. Thin air at Ladakh
D. High albedo of ice as compared to regions down the hill
User Answer : C
Correct Answer : C
Answer Justification :
Justification: The altitude in Ladakh varies from about 3000m (in Kargil) to more than 8,000m (in the
Karakoram).
Due to such a high altitude, the climate is extremely cold and dry. So, option A and B are wrong.
Option D is also wrong because albedo of Ice reflects the sunlight, whereas here we are talking about direct
rays of the Sun.
The air at this altitude is so thin that the heat of the sun can be felt intensely.
The area experiences freezing winds and burning hot sunlight.
You will be surprised to know that if you sit in the sun will your feet in the shade, you may suffer from both
sunstroke and frost bite at the same time.
Learning: The day temperatures in summer are just above zero degree and the night temperatures well below
30C. It is freezing cold in the winters when the temperatures may remain below 40C for most of the time. As it
lies in the rain shadow of the Himalayas, there is little rainfall, as low as 10 cm every year.
Q Source: Improvisation: Page 74: Geography NCERT 7th: Our Environment

52. Kuji Mendha is a breed of sheep found only in coastal Odisha. Why it was recently conferred rare and singular
species tag by the Union government?
A. It was declared Near Extinction in the IUCN Red List.
B. It has some rare genetic traits which lead to multiple birth syndromes in it.
C. It survives in the harshest conditions known to the breed of this species and still highly productive in terms
of wool and meat.
D. All of the above
User Answer : B
Correct Answer : B
Answer Justification :
Justification: Scientific studies on this rare breed found the sheep to be carrying a rare gene mutation.
Kuji Mendha sheep are fast breeders giving multiple birth.
Sheep in other parts of Odisha are not known for giving multiple birth. This characteristic makes them
distinctive from other species.
(C) Insights Active Learning. | All rights reserved.

www.insightsias.com

31

TEST - 1

Total Marks :

200

Mark Scored :

110.67

Learning: They are well adapted to high ambient temperature, high humidity and heavy rains.
They are found in Jagatsinghpur and Kendrapara districts of Odisha.
The National Bureau of Animal Genetic Resources (NBAGR) has accorded the genetic recognition.
With NBAGR conferring it genetically rare status, conserving these domesticated species would receive a
boost.
Q
Source:http://timesofindia.indiatimes.com/home/environment/flora-fauna/Kendrapara-sheep-gets-rare-status/articl
eshow/53140281.cms

53. Consider the following sources of freshwater on earth.


1.
2.
3.
4.
5.

Groundwater
Atmosphere
Ice caps
Fresh water lakes
Rivers
Arrange the following in increasing order of availability of freshwater from the codes given below.
A. 2<1<4<5<3
B. 2<4<1<3<5
C. 1<2<4<5<3
D. 5<2<4<1<3
User Answer : A
Correct Answer : D
Answer Justification :
Justification: You should first apply elimination in such questions. It is common knowledge that ice-caps are the
largest source of fresh water, which eliminates B.
Also, since groundwater is more abundant than river water, it eliminates C and A.
The clear answer is D.
Distribution ofwater as given in NCERT.

It can be understood by an analogy.


(C) Insights Active Learning. | All rights reserved.

www.insightsias.com

32

TEST - 1

Total Marks :

200

Mark Scored :

110.67

Q Source: Page 32: Geography NCERT 7th: Our Environment

54. The Ministry of Environment, Forest and Climate Change (MoEFCC) has suggested that it will propose for the
funding of foam industries to shift to cyclopentane. How the shift to cyclopentane will be useful?
1. It has zero ozone depletion potential and low global warming potential.
2. It has very low flammability and will help avoid fire hazards in industries.
Which of the above is/are correct?
A. 1 only
B. 2 only
C. Both 1 and 2
D. None
User Answer : A
Correct Answer : A
Answer Justification :
Justification: Statement 1: The shift will be made from Hydrochlorofluorocarbons (HCFCs) which are ozone
depleting.
HCFC is a commonly used refrigerant gas. It is also used in foam blowing agents, solvents, aerosols and
fire extinguishers.
India is in the process of phasing out HCFC by 2030 as per the existing agreement signed by India in the
Montreal protocol.
The transition will be funded by the Executive Committee of the Multilateral fund as per the Montreal
Protocol Agreement in the second stage of HCFC phase out. SMEs will encouraged to shift to
cyclopentanes.
Statement 2: Cyclopentane is a cheaper alternative (HCFC is patented and expensive) without any patents
associated with it.
But, some SMEs also expressed their reservations against using cyclopentane due to its flammability and the need
for skilled technicians to handle it.

(C) Insights Active Learning. | All rights reserved.

www.insightsias.com

33

TEST - 1

Total Marks :

200

Mark Scored :

110.67

Q Source:
http://www.downtoearth.org.in/news/india-decides-to-put-a-break-on-hfo-in-foam-sector-for-now-55227

55. Consider the following about the Directorate of Enforcement.


1. It is housed under the Ministry of Corporate Affairs.
2. It enforces the Prevention of Money Laundering Act.
3. It keeps track of foreign exchange movement in and out of India.
Select the correct answer using the codes below.
A. 1 and 2 only
B. 2 and 3 only
C. 1 only
D. 2 only
User Answer : B
Correct Answer : B
Answer Justification :
Justification: Statement 1: Directorate of Enforcement is a specialized financial investigation agency under the
Department of Revenue, Ministry of Finance. So, 1 is wrong.
Statement 2 and 3: It enforces the Foreign Exchange Management Act,1999 (FEMA) and Prevention of Money
Laundering Act, 2002 (PMLA).
The Act and Rules notified there under impose obligation on banking companies, financial institutions and
intermediaries to verify identity of clients, maintain records and furnish information
You can read more about the laws here at the Q Source.
Q Source: http://www.insightsonindia.com/2016/08/02/insights-daily-current-affairs-02-august-2016/

56. Impeyan monal is most likely to be found in its natural habitat in


A. Adjoining oases of the hot deserts of Rajasthan
B. Swamp forests of Andaman & Nicobar Islands
C. Temperate forests in the Himalayan region
D. Salt marshes of Gujarat
User Answer : C
Correct Answer : C
Answer Justification :
Learning: Himalayan monal is also known as the Impeyan monal or pheasant is state bird of Uttarakhand and
national bird of Nepal, where it is known as Danphe. It was also the state bird of Himachal Pradesh, until 2007.
The bird's natural range extends from eastern Afghanistan through the Himalayas in Pakistan, Kashmir
region and the Republic of India (states of Himachal Pradesh, Uttarakhand, Sikkim and Arunachal Pradesh),
Nepal, southern Tibet, and Bhutan.
It occupies upper temperate oak-conifer forests interspersed with open grassy slopes, cliffs and alpine
meadows
In some areas, the species is threatened due to poaching and other anthropogenic factors.

(C) Insights Active Learning. | All rights reserved.

www.insightsias.com

34

TEST - 1

Total Marks :

200

Mark Scored :

110.67

Q Source: Improvisation: Page 42: Fig 6.10: Geography NCERT 7th: Our Environment

57. Which of the following differentiates correctly between Tropical Savannahs and Temperate Grasslands?
A. Temperate grass is longer but much less nutritious than those of Tropical Savannas.
B. Temperate grasslands are drier and receive lesser rainfall than savannas.
C. Temperate Grasslands have scattered trees, unlike in Savannas where the only vegetation is grass and
mosses.
D. Temperate grasslands are found near the coastal regions, whereas Tropical Savannas occur in interior areas
in Northern Hemisphere only.
User Answer : B
Correct Answer : B
Answer Justification :
Justification:Option A: Temperate grass here is short and nutritious, not longer than Savannah. So, A is wrong.
Option B & C: Savannah is Grassland characterized by scattered trees that are not able to form a canopy. As there
is no canopy, the surface gets sufficient light, and the area supports grasses.
Temperate grasslands are characterized by grasses of different types with no trees and only a few types of shrubs.
The reason of no trees lies in the fact that temperate grasslands receive lesser rainfall and are drier. So, C is wrong
and B is correct.
Option D: These grow on either side of the equator and extend till the tropics. This vegetation grows in the areas
of moderate to low amount of rainfall.
Temperate Grasslands are found in the mid-latitudinal zones and in the interior part of the continents. So, D is
wrong.
Q Source: Improvisation: Page 43: Geography NCERT 7th: Our Environment

(C) Insights Active Learning. | All rights reserved.

www.insightsias.com

35

TEST - 1

Total Marks :

200

Mark Scored :

110.67

58. Apart from Wheat, Crops that are usually grown in the plains of India are
1.
2.
3.
4.
5.

Maize
Sorghum
Coffee
Millets
Sugarcane
Select the correct answer using the codes below.
A. 1 and 5 only
B. 1, 2, 4 and 5 only
C. 2, 3 and 4 only
D. 1, 2, 3, 4 and 5
User Answer : A
Correct Answer : B
Answer Justification :
Justification: The soil can be fertile in plain areas. This is ideal for growing crops like Sugarcane, Jute, sorghum,
rice, wheat etc.
Millets can even grow in relatively drier areas with less fertile soils.
Banana plantations are seen in some areas of the plain.
Coffee and tea are generally grown in hilly areas. So, 3 is wrong.
Q Source: Page 60: Geography NCERT 7th: Our Environment

59. The Buddha taught that human life is full of suffering and unhappiness, which is caused due to
A. Separation of matter with soul
B. Law of Karma
C. Constant cravings and desires
D. Following the middle way in everything which dilutes focus and energy
User Answer : C
Correct Answer : C
Answer Justification :
Justification: The suffering is caused because we have cravings and desires (which often cannot be fulfilled).
Sometimes, even if we get what we want, we are not satisfied, and want even more (or want other things).
The Buddha described this as thirst or tanha.
He taught that this constant craving could be removed by following moderation in everything. So, clearly
(d) is wrong.
Buddha doesnt pin point the law of karma to be responsible for our suffering. He only believed that the
results of our actions (called karma), whether good or bad, affect us both in this life and the next (some
authorities deny this aspect of Buddhism). So, (b) cant be correct, if option (c) is already mentioned as later
is more appropriate.
Learning: He also taught people to be kind, and to respect the lives of others, including animals.
The Buddha taught in the language of the ordinary people, Prakrit, so that everybody could understand his
message.
Q Source: Improvisation: Page 66: NCERT 6th History: Our Pasts-I
(C) Insights Active Learning. | All rights reserved.

www.insightsias.com

36

TEST - 1

Total Marks :

200

Mark Scored :

110.67

60. Consider the following locations in Northern India.


1.
2.
3.
4.

Dehra Gompas
Siachen Glacier
Rohtang pass
Zoji la pass
The correct order of these regions from North to South is?
A. 2143
B. 2314
C. 1342
D. 3241
User Answer : B
Correct Answer : A
Answer Justification :
Justification: You should use elimination.
You must have heard of the Manali to Rohtang pass way. On the other hand, Siachen Glacier is known be located
in Northern J&K. This implies that Rohtang pass (statement 3) must lie to the South of Siachen Glacier (statement
2).
So, C and D can be easily eliminated.
If you know that Zoji la pass lies between Leh and Srinagar, it implies that Zoji la must lie to the North of Rohtang
pass. So, the only answer possible is option A.
Learning: Manali - Leh highway crosses four passes, Rohtang la, Baralacha la Lungalacha la and Tanglang la.

Q Source: Fig 10.4: Page 74: Geography NCERT 7th: Our Environment
(C) Insights Active Learning. | All rights reserved.

www.insightsias.com

37

TEST - 1

Total Marks :

200

Mark Scored :

110.67

61. Consider the following statements.


1. The air pressure is highest at sea level and decreases with height.
2. Low air pressure is generally associated with cloudy skies and wet weather.
Which of the above is/are correct?
A. 1 only
B. 2 only
C. Both 1 and 2
D. None
User Answer : C
Correct Answer : C
Answer Justification :
Justification: Statement 1: Air pressure is defined as the pressure exerted by the weight of air on the earths
surface. As we go up the layers of atmosphere, the pressure falls rapidly.
Statement 2: In areas where temperature is high the air gets heated and rises. This creates a low-pressure area.
Low pressure is associated with cloudy skies and wet weather.
In areas having lower temperature, the air is cold. It is therefore heavy.
Heavy air sinks and creates a high pressure area. High pressure is associated with clear and sunny skies.
The air always moves from high pressure areas to low pressure areas.
Q Source: Page 24: Geography NCERT 7th: Our Environment

62. Factors that affect the formation of tides in the Ocean are
1.
2.
3.
4.

Alignment of Earth, Sun and Moon.


Relative distance between the Moon, Earth and Sun .
Shape of bays and estuaries where tides are formed.
Local wind and weather patterns in Ocean.
Select the correct answer using the codes below.
A. 1 and 2 only
B. 2 and 3 only
C. 1 and 4 only
D. 1, 2, 3 and 4
User Answer : A
Correct Answer : D
Answer Justification :
Justification: Statement 1 and 2: The strong gravitational pull exerted by the sun and the moon on the earths
surface causes the tides.
The water of the earth closer to the moon gets pulled under the influence of the moons gravitational force and
causes high tide.
During the full moon and new moon days, the sun, the moon and the earth are in the same line and the tides
are highest. These tides are called spring tides.
But when the moon is in its first and last quarter, the ocean waters get drawn in diagonally opposite
directions by the gravitational pull of sun and earth resulting in low tides. These tides are called neap tides.
Statement 3: The shape of bays and estuaries also can magnify the intensity of tides. Funnel-shaped bays in
particular can dramatically alter tidal magnitude.

(C) Insights Active Learning. | All rights reserved.

www.insightsias.com

38

TEST - 1

Total Marks :

200

Mark Scored :

110.67

The Bay of Fundy in Nova Scotia is the classic example of this effect, and has the highest tides in the world
- over 15 meters.
Narrow inlets and shallow water also tend to dissipate incoming tides.
In estuaries with strong tidal rivers, such as the Delaware River and Columbia River, powerful seasonal
river flows in the spring can severely alter or mask the incoming tide.
Statement 4: Strong offshore winds can move water away from coastlines, exaggerating low tide exposures.
Onshore winds may act to pile up water onto the shoreline, virtually eliminating low tide exposures.
High-pressure systems can depress sea levels, leading to clear sunny days with exceptionally low tides.
Conversely, low-pressure systems that contribute to cloudy, rainy conditions typically are associated with
tides than are much higher than predicted.
Q Source: Improvisation: Page 36: Geography NCERT 7th: Our Environment

63. Anantnag, Baramula, and Pahalgam are well known for


A. Production of Kashmiri willow Cricket Bats
B. Cave paintings of Sonbhadra and Chitrakoot
C. Fabrics like chiffon, muslin and organza
D. Metalwork of Badohi and decorated terracotta horse
User Answer : A
Correct Answer : A
Answer Justification :
Justification:Option A: These are some of the districts where Kasmir willow bats are made. The willow used in
making these bats was brought in by the British. These willows are lighter and more powerful than ordinary wood,
making them more effective as a hitting bat.
Option B: The cave paintings of Sonbhadra and Chitrakoot depict scenes of hunting, war, festivals, dances,
romantic life and animals and belong to UP.
Option C: One of the important crafts of Uttar Pradesh is Chikankari, which entails delicate and traditional hand
embroidery. This form of handicrafts is mainly practiced in Lucknow. It is done on fabrics like chiffon, muslin,
organza, organdie and silk.
Option D: Gorakhpur has villages where clay figures of animals are done and It is famous for its ornately
decorated terracotta horse. The potter creates the basic form by throwing separate pieces on the wheel and then
joining them.
Moradabad in U P is famous for art metalwork and known for its coloured enamelling and intricate engravings.
Q Source: Improvisation: Page 75: Geography NCERT 7th: Our Environment

64. Recently, India received the official citation from WHO and UNICEF for being YAWS-free. Consider the following
with reference to this.
1. The Disease affects mainly the old age people due to lack of proper immune response from the body.
2. India is the first country to be officially acknowledged as being Yaws-free.
Which of the above is/are correct?
A. 1 only
B. 2 only
C. Both 1 and 2
(C) Insights Active Learning. | All rights reserved.

www.insightsias.com

39

TEST - 1

Total Marks :

200

Mark Scored :

110.67

D. None
User Answer :
Correct Answer : B
Answer Justification :
Justification: Statement 1: It affects mainly children before the age of 15 years. 75-80% cases are related to
children.
The disease occurs mainly in poor communities in warm, humid, tropical areas where lack of proper water and
sanitation facilities further help the disease spread.
Statement 2: India has achieved this important milestone of being Yaws-free much before the WHO global target
year of 2020.
Learning: Yaws is a chronic infection that affects mainly the skin, bone and cartilage.
It is caused by the bacterium - Treponema pallidumm and transmitted by skin contact.
Eradication of Yaws in India has been possible because of education and early treatment of vulnerable population.
You can read more about it here http://www.who.int/mediacentre/factsheets/fs316/en/
Q Source: Current Affairs: WHO Website:
http://www.who.int/neglected_diseases/news/India_global_eradication_yaws_celebration/en/

65. What is the Green corridor project that is recently being pushed by state governments?
A. A project for responding to climate change by a combination of adaptation and mitigation measures
including carbon sequestration.
B. A dedicated State Investment and Manufacturing Zone (SIMZ) for establishment of Green Energy
companies and services.
C. A project for evacuation of renewable energy from generation points to the load centres by creating
intrastate and inter-state transmission infrastructure.
D. A project for protecting, restoring and enhancing Indias diminishing forest cover by establishing inter-state
buffer protected areas.
User Answer : B
Correct Answer : C
Answer Justification :
Justification: Green Energy Corridor Project aims at synchronising electricity produced from renewable sources,
such as solar and wind, with conventional power stations in the grid.
The government has taken lessons from the massive power grid failure that hit the North, East and NorthEast regions of the country in 2012.
Germany, who has expertise in making smart grids that integrate renewable energy into national grid will be
assisting India in this project.
The intra-state transmission component of the project is being implemented by the respective states and the
Power Grid Corporation of India (PGCIL) is implementing inter-state component.
Learning: PGCIL is setting up the first corridor connecting states rich in renewable energy.
A second corridor would connect the solar parks in Andhra Pradesh, Madhya Pradesh, Karnataka, Rajasthan
and Gujarat.
The problem of voltage fluctuation in integrating conventional grids with renewable energy grids would be
(C) Insights Active Learning. | All rights reserved.

www.insightsias.com

40

TEST - 1

Total Marks :

200

Mark Scored :

110.67

taken care of by the German technology and support.


Q
Source:http://www.thehindu.com/news/national/tamil-nadu/indias-first-green-corridor-inaugurated/article8895647
.ece

66. Which of the following are Permanent winds on the planet earth?
A. Easterlies ad Westerlies
B. Monsoon winds
C. Continental winds
D. Orographic winds
User Answer : A
Correct Answer : A
Answer Justification :
Learning: Winds can be broadly divided into three types.
Permanent winds The trade winds, westerlies and easterlies are the permanent winds.
These blow constantly throughout the year in a particular direction.
Seasonal winds These winds change their direction in different seasons. For example monsoons in India.
Local winds These blow only during a particular period of the day or year in a small area. For example,
land and sea breeze, and loo.

Q Source: Page 25: Geography NCERT 7th: Our Environment

67. Which of the following statements about the Troposphere is/are NOT correct?
A. Almost all the weather phenomena like rainfall and hailstorm occur in this layer.
B. It is the most ideal atmospheric layer for flying aeroplanes.
C. It helps in radio transmission as radio waves transmitted from the earth are reflected back to the earth by
(C) Insights Active Learning. | All rights reserved.

www.insightsias.com

41

TEST - 1

Total Marks :

200

Mark Scored :

110.67

this layer.
D. Both (b) and (c)
User Answer : D
Correct Answer : D
Answer Justification :
Justification:Option A: Clouds and most other gases occur in this layer. Movement of air mass (convection
currents) within the layer gives rise to weather phenomena.
Option B: It is stratosphere as this layer is almost free from clouds and associated weather phenomenon, making
conditions most ideal for flying aeroplanes.
Option C: It is ionosphere which extends from 80-400 Km in height.
Learning:Exosphere: The upper most layer of the atmosphere is known as exosphere. This layer has very thin air.
Light gases like helium and hydrogen float into the space from here.
Q Source: Page 23: Geography NCERT 7th: Our Environment

68. Many states in the Indian Union have instituted the post of Parliamentary Secretary. A Parliament Secretary is
1. Generally a Civil Servant/bureaucrat holding an Office of Profit
2. Assigned charge of an entire government department only in the absence of Cabinet Ministers
Which of the above is/are correct?
A. 1 only
B. 2 only
C. Both 1 and 2
D. None
User Answer : D
Correct Answer : D
Answer Justification :
Justification & Learning: A Parliamentary Secretary often holds the rank of Minister of State and has the same
entitlements and is assigned to a government department.
Manipur, HP, Mizoram, Assam, Rajasthan, Punjab, Goa are some of the states where MLAs have been
appointed Parliament Secretaries by the Government.
Various petitions in the High Court have challenged the appointment of Parliament Secretary, including in
Delhi most recently.
In June 2015, Calcutta HC quashed appointment of 24 Parliamentary Secretaries in West Bengal dubbing it
unconstitutional.
Delhi High Court in 2016 quashed the appointment of Parliamentary secretaries as they are deemed to be
Ministers, and there is cap on the number of ministers in the government as per the Constitution (The limit is 10%
for Delhi, owing to its special status)
Q
Source:http://timesofindia.indiatimes.com/india/Parliamentary-secretaries-de-facto-ministers-Law-ministry/article
show/52755852.cms

69. The campaign 'Swachh Yug' launched by the government aims at


(C) Insights Active Learning. | All rights reserved.

www.insightsias.com

42

TEST - 1

Total Marks :

200

Mark Scored :

110.67

A. Making Gram Panchayats located along Ganga open defecation-free .


B. Rehabilitating manual scavengers in rural areas.
C. Uplifting the moral standards in school and college education.
D. Increasing the participation of youth and children in Swacch Bharat Mission all over India.
User Answer :
Correct Answer : A
Answer Justification :
Learning: It is a collaborative effort of three Union Ministries, to bring about behavioural change among people
staying in villages along the river Ganga.
It covers five states - UP, Uttarakhand, Bihar, Jharkhand and West Bengal from where the river flows.
A nodal officer has been identified for each district to work on making area under their jurisdiction open
defecation free (ODF) in a "mission mode" and clean through proper solid and liquid waste management.
In addition to monetary incentive offered under Swachh Bharat Mission, extensive interpersonal behaviour
change communication training will be given to local trainers through network of virtual classrooms.
Ministries involved in campaign
Ministry of Drinking Water and Sanitation mission mode strategy to focus on cleanliness of village
through proper solid and liquid waste management.
Ministry of Youth Affairs and Sports will enlist support of youth agencies like Bharat Scouts and Guides,
Nehru Yuva Kendras and National Service Scheme.
Ministry of Water Resources, River Development and Ganga Rejuvenation
Q Source:http://pib.nic.in/newsite/PrintRelease.aspx?relid=145999

70. The Jatakas often shown on the railings of stupas and in paintings in places such as Ajanta are actually?
A. Stories that tell about the previous lives of the Buddha.
B. Essential teachings of the Buddha and his disciples.
C. Cycles of birth and death as deciphered by the monks.
D. Sangha rules that helped them travel and live together to propagate dhamma.
User Answer : A
Correct Answer : A
Answer Justification :
Justification: Sangha rules are actually written in Vinaya pitaka, so D is incorrect.
Buddhas teachings are given in Sutta Pitaka. So, B is incorrect.
Abhidhammapitaka deals with the philosophy and doctrine of Buddhism appearing in the suttas.
Learning: Jataka stories are preserved in all branches of Buddhism. Some Jataka tales are scattered in various
sections of the Pali canon of Buddhist writings,
The future Buddha may appear in them as a king, an outcast, a god, an elephant - but, in whatever form, he
exhibits some virtue that the tale thereby inculcates.
Many Jatakas have parallels in the Mahabharata, the Panchatantra (animal fables), the Puranas, and
elsewhere in non-Buddhist Indian literature.
Q Source: Page 129: NCERT 6th History: Our Pasts-I

71. Consider the following about Glaciers.


(C) Insights Active Learning. | All rights reserved.

www.insightsias.com

43

TEST - 1

Total Marks :

200

Mark Scored :

110.67

1. Glacial retreat can result in variety of landforms like lakes.


2. Materials deposited by a glacier as it retreats results inglacial moraines.
Which of the above is/are correct?
A. 1 only
B. 2 only
C. Both 1 and 2
D. None
User Answer : C
Correct Answer : C
Answer Justification :
Justification: Statement 1: The material carried by the glacier such as rocks big and small, sand and silt gets
deposited. These deposits form glacial moraines.
Statement 2: Glaciers movements carve out deep hollows. When they retreat and as the ice melts these hollows
get filled up with water and become beautiful lakes in the mountains.
You can read all about Glaciers and their features here http://nationalgeographic.org/encyclopedia/glacier/
Q Source: Improvisation: Page 16: Geography NCERT 7th: Our Environment

72. Krishi Kalyan Cess, to finance improvement of agriculture and farmers welfare, is levied in India on
A. Services sector
B. Manufacturing Sector
C. Fertilizers and natural gas sector
D. Groundwater irrigation equipments
User Answer : A
Correct Answer : A
Answer Justification :
Learning: Imposition of Krishi Kalyan Cess of 0.5% was announced in the Budget 2016-17 to come to force on
1st June 2016.
KKC is a cess, applicable on all services. It is to be solely used towards financing activities for the
improvement of agriculture and farmer welfare.
Hence new effective Service Tax hence increased to 15% from 14.5%.
Budget target for Krishi Kalyan Cess is Rs. 5000 crore.
The KKC is based on the idea that levying a cess on a thriving sector service sector (58% of GDP) of the
economy can help fund a lifeline to a sector that is in distress (Agriculture).
Q
Source:http://indianexpress.com/article/business/budget/budget-2016-govt-announces-0-5-pc-krishi-kalyan-cess-t
o-fund-farm-programmes/

73. A loan has been approved by an international agency for constructing a bridge across Ganga in Bihar. How will the
Ganga Bridge be helpful?
1. It will provide vital transport link between Southern Uttar Pradesh and Odisha.
2. It will improve Bihars connectivity with Nepal.
Which of the above is/are correct?
A. 1 only
B. 2 only
(C) Insights Active Learning. | All rights reserved.

www.insightsias.com

44

TEST - 1

Total Marks :

200

Mark Scored :

110.67

C. Both 1 and 2
D. None
User Answer :
Correct Answer : B
Answer Justification :
Justification: Statement 1 and 2: The bridge would provide vital transport links between the northern and
southern parts of the state and with neighbouring country Nepal.
The Ganga railroad bridge connects Digha Ghat in Patna and Pahleja Ghat in Sonpur, Saran district in
Bihar.
Regular scheduled passenger rail service was inaugurated on this route on 3 February 2016.
This is second railway bridge in Bihar after Rajendra Setu that connects North Bihar to South Bihar
Learning: Asian Development Bank (ADB) has recently approved $500 million (approximately Rs 3350 crore)
loan for the bridge.
Once built, the 9.8 km road bridge in Bihar will be countrys longest river bridge.
The project will run for about 4 years and is expected to complete by the end of December 2020.
Q
Source:http://www.hindustantimes.com/patna/foundation-stone-for-bihar-bridge-laid-again-for-lalu-s-sake/story-n
EVZIGu0LWETQjQo20yaBP.html

74. Consider the following about the Asian Development Bank (ADB).
1. It was founded before World Bank (WB).
2. ADB assists its members by providing loans and technical assistance to promote socio-economic development.
3. All its members are from the Asia-Pacific region.
Select the correct answer using the codes below.
A. 1 only
B. 2 and 3 only
C. 2 only
D. 1 and 3 only
User Answer : C
Correct Answer : C
Answer Justification :
Justification: Statement 1: The Asian Development Bank was conceived in the early 1960s as a financial
institution that would be Asian in character and foster economic growth and cooperation in one of the poorest
regions in the world.
Statement 2: ADB assists its members (for e.g. India in infrastructure projects), and partners, by providing loans,
technical assistance, grants, and equity investments to promote social and economic development.
Statement 3: ADB is composed of 67 members, 48 of which are from the Asia and Pacific region. So, 3 is wrong.
India became a member of the Asian Development Bank (ADB) as a founding member in 1966.
Japan and the US represent the largest shareholders.
Q Source: Ganga bridge loan by ADB
(C) Insights Active Learning. | All rights reserved.

www.insightsias.com

45

TEST - 1

Total Marks :

200

Mark Scored :

110.67

75. Consider the following about the Nano Mission of the Government of India.
1. The Department of Science and Technology is the nodal agency for implementing the Nano Mission.
2. It can publish guidelines on Safe Handling of Nano-materials.
3. It funds all the private research agencies on development of nano-materials.
Select the correct answer using the codes below.
A. 1 and 2 only
B. 2 and 3 only
C. 3 only
D. 1 and 3 only
User Answer :
Correct Answer : A
Answer Justification :
Justification: Statement 1: The right answer can be reached by elimination easily.
The Government of India, in 2007, had approved the launch of a Mission on Nano Science and Technology (Nano
Mission) with DST as the nodal body.
Statement 2: The Nano Mission has recently come out with the draft Guidelines and Best Practices for Safe
Handling of Nanomaterials in Research Laboratories and Industries.
Statement 3: The Nano Mission will strive for development of products and processes for national development,
especially in areas of national relevance like safe drinking water, materials development, sensors development,
drug delivery, etc.
It doesnt fund all private research in India. So, 3 is wrong.
We will be covering questions on the applications of nanotechnology in the coming tests.
Q Source:http://dst.gov.in/scientific-programmes/mission-nano-science-and-technology-nano-mission

76. Consider the following statements about the Geography of continents.


1. Asia is separated from Europe by the Alps Mountains on the west.
2. The Arctic Circle passes through Europe.
3. North America is linked to South America by a narrow strait of land.
Select the correct answer using the codes below.
A. 1 and 2 only
B. 2 and 3 only
C. 1 and 3 only
D. 1, 2 and 3
User Answer : B
Correct Answer : B
Answer Justification :
Justification: Statement 1: The diagram below shows it. It is Ural mountains.

(C) Insights Active Learning. | All rights reserved.

www.insightsias.com

46

TEST - 1

Total Marks :

200

Mark Scored :

110.67

Statement 2:

Statement 3: It is the Isthmus of Panama that connects North with South America.
Q Source: Page 32-33: NCERT 6th Geography: The Earth Our Habitat

77. Dhamma Mahamattas were appointed by Ashoka to


A. Design a new religious code for the public
B. Administer all donations to religious organizations and trusts in the empire
C. Teach people about Ashokas Dhamma
D. Represent the empire in religious debates with other empires and eminent scholars
User Answer : C
Correct Answer : C
Answer Justification :

78. Carbfix project is concerned with


A. Unearthing gas hydrates trapped at the Ocean floor
B. Locking away atmospheric CO2 by reacting it with basaltic rocks
C. Reducing the methane emissions of rice fields and wetlands
(C) Insights Active Learning. | All rights reserved.

www.insightsias.com

47

TEST - 1

Total Marks :

200

Mark Scored :

110.67

D. Using geo-engineering to reduce the albedo of Polar Ice


User Answer : B
Correct Answer : B
Answer Justification :
Justification: It is a project in Iceland that aims to lock away CO2 by reacting it with basaltic rocks.
Carbonated water is injected into the rocks so that it reacts with Calcium, Magnesium or Silicate material present
in Basaltic rocks. This is called enhanced weathering.
Thus, the CO2 is captured permanently without releasing any harmful by-products.
Learning: Recent reports show that the project was able to solidify 95% of the injected 250 tonnes of CO2 into
calcite in 2 years, using 25 tonnes of water per tonne of CO2.
This is a significant achievement and offers hope for future.
Cost of the process is very high. Also, since the reactions are exothermic; it is reversible if the rocks are
heated.
Further, the pumping activity generates seismic activity.
Q
Source:http://www.thehindu.com/sci-tech/science/scientists-turn-co2-into-rock-to-combat-climate-change/article8
715026.ece

79. In the context of Ancient history of India, Gramabhojakas and Grihapattis respectively were
A. Largest landlords and Head Judge of the village
B. Slaves and bonded labourers
C. Census official and Policeman of the village
D. Village headman and independent farmers
User Answer : D
Correct Answer : D
Answer Justification :
Justification & Learning: In the northern part of the country, the village headman was known as the grama
bhojaka.
Usually, men from the same family held the position for generations (post was hereditary).
The grama bhojaka was often the largest landowner. Generally, he had slaves and hired workers to cultivate
the land.
Besides, as he was powerful, the king often used him to collect taxes from the village. He also functioned as
a judge, and sometimes as a policeman.
Apart from the gramabhojaka, there were other independent farmers, known as grihapatis, most of whom
were smaller landowners.
And then there were men and women such as the dasa karmakara, who did not own land, and had to earn a
living working on the fields owned by others.
Q Source: Page 89: NCERT 6th History: Our Pasts-I

80. The Hubble sequence is


A. The order of evolution of life on earth after the Big Bang
(C) Insights Active Learning. | All rights reserved.

www.insightsias.com

48

TEST - 1

Total Marks :

200

Mark Scored :

110.67

B. A system used for classifying galaxies


C. A code used by the International Space Station to communicate back to earth
D. Made of all stars within the Goldilocks zone
User Answer : C
Correct Answer : B
Answer Justification :
Learning: Hubbles scheme divides regular galaxies into 3 broad classes ellipticals, lenticulars and spirals
based on their visual appearance.
Although this scheme, also known as the Hubble tuning fork diagram, is now considered somewhat too simple, the
basic ideas still hold.
For a time the Hubble tuning fork was thought to be an evolutionary sequence - that galaxies might evolve from
one type to another progressing from left to right across the tuning-fork diagram.

Q Source: Current Affairs: NASA Website:


https://www.nasa.gov/image-feature/goddard/2016/nasas-hubble-spots-a-lopsided-lynx

81. Consider the following statements.


1. Warm ocean currents generally originate near sub-tropical regions and move towards equator.
2. The cold currents carry water from polar or higher latitudes to lower latitudes.
3. The areas where the warm and cold currents meet are devoid of nutrient cycling and thus generally poor in
fisheries.
Select the correct answer using the codes below.
A. 1 and 2 only
B. 2 only
C. 1 and 3 only
D. 1, 2 and 3
User Answer : B
Correct Answer : B
Answer Justification :
Justification: Statement 1 and 2: Generally, the warm ocean currents originate near the equator and move
towards the poles. So, 1 is wrong.
(C) Insights Active Learning. | All rights reserved.

www.insightsias.com

49

TEST - 1

Total Marks :

200

Mark Scored :

110.67

The cold currents carry water from polar or higher latitudes to tropical or lower latitudes. So, 2 is correct.
The Labrador Ocean current is cold current while the Gulf Stream is a warm current.
The ocean current influence the temperature conditions of the area. Warm currents bring about warm
temperature over land surface, and the opposite is also true.
Statement 3: They are the best fishing grounds due to recycling of nutrients. Seas around Japan and the eastern
coast of North America are such examples. The areas where a warm and cold current meet also experience foggy
weather making it difficult for navigation.
Q Source: Page 36-37: Geography NCERT 7th: Our Environment

82. The SDR is an international reserve asset, created by the IMF in 1969 to supplement its member countries official
reserves. Consider the following about Special Drawing Rights (SDR).
1. SDRs cannot be exchanged for freely usable currencies.
2. Value of a SDRis fixed on an annual basis and revised on the discretion of IMF.
Which of the above is/are correct?
A. 1 only
B. 2 only
C. Both 1 and 2
D. None
User Answer :
Correct Answer : D
Answer Justification :
Justification: Statement 1: It was created as a supplementary international reserve asset.
SDRs can be exchanged for freely usable currencies. So, 1 is wrong.
The value of the SDR is currently based on a basket of four major currencies: the U.S. dollar, euro, the Japanese
yen, and pound sterling. The basket will be expanded to include the Chinese renminbi (RMB) as the fifth currency,
effective October 1, 2016.
Statement 2: Its value is determined daily based on the currency values of its constituent currencies.
The SDR is neither a currency, nor a claim on the IMF. Rather, it is a potential claim on the freely usable
currencies of IMF members.
Q Source: Often in news as it new SDR will be effective from October 2016

83. The Painted Grey Ware culture (PGW) is an


A. Pre-Vedic age culture
B. Iron Age culture
C. Stone Age culture
D. Soanian culture
User Answer : B
Correct Answer : B
Answer Justification :
Learning: It is an Iron Age culture of the Gangetic plain and the Ghaggar-Hakra valley, lasting from roughly 1200
BCE to 600 BCE.
PGW are extremely fine to touch, with a nice, smooth surface. Perhaps these were used on special
(C) Insights Active Learning. | All rights reserved.

www.insightsias.com

50

TEST - 1

Total Marks :

200

Mark Scored :

110.67

occasions, for important people, and to serve special food.


It is contemporary to, and a successor of the Black and red ware culture.
It is succeeded by Northern Black Polished Ware from c. 700-500 BCE, associated with the rise of the great
mahajanapada states and of the Magadha Empire.

Q Source: Page 56: NCERT 6th History: Our Pasts-I

84. The Financial Stability Board (FSB) is an international body that monitors and makes recommendations about the
global financial system. Consider the following about it.
1. It was established by G-20 as the successor to the Financial Stability Forum (FSF).
2. It has been made a member organ of the International Monetary and Financial Committee (IMFC).
Which of the above is/are correct?
A. 1 only
B. 2 only
C. Both 1 and 2
D. None
User Answer :
Correct Answer : A
Answer Justification :
Justification: Statement 1: To strengthen the surveillance of financial markets, the G20 leaders decided in April
2009 to expand the membership of the former Financial Stability Forum (FSF) and renamed it the Financial
Stability Board (FSB). The new membership includes all G20 countries, Hong Kong SAR, Singapore, Spain, and
the European Commission.
Statement 2: FSB will report any possible threats to the stability of the global financial system to the G20 finance
ministers, the IMF and central bank governors.
The FSF was made an observer of the IMFC in 1999. FSB is not a member organ of the IMFC.
(C) Insights Active Learning. | All rights reserved.

www.insightsias.com

51

TEST - 1

Total Marks :

200

Mark Scored :

110.67

You can read more about it here http://www.fsb.org/about/


Q Source: Often in news: http://www.imf.org/external/np/exr/facts/groups.htm#IC

85. South west monsoon season is marked by the onset and advance of monsoon. In this season the Monsoon winds
blow from
A. Arabian Sea and Bay of Bengal towards the land
B. Only from Central Pacific towards Bay of Bengal
C. Land to the Bay of Bengal and Southern India
D. Only from Arabian Sea towards the land
User Answer : A
Correct Answer : A
Answer Justification :
Justification: These Monsoon winds lift moisture from both these oceans and carry moisture with them.
When these winds strike the mountain barriers (say in Himalayas or North-eastern Mountains), rainfall occurs.
We will be covering other patterns of wind and ocean current circulation like El-Nino (that affect Monsoon) in
later tests.
Q Source: Page 57: NCERT 6th Geography: The Earth Our Habitat

86. If the earth starts rotating east to west, which of the following shall be correct?
1. Places West of Greenwich will be behind the Greenwich Time.
2. Places located on the same longitude northwards will be ahead in time than those located southwards.
Which of the above is/are correct?
A. 1 only
B. 2 only
C. Both 1 and 2
D. None
User Answer : D
Correct Answer : D
Answer Justification :
Justification: Statement 1: It is the case presently when earth rotates from west to east. If the opposite happens,
places west of Greenwich will be ahead in time and places east will lag behind in time.
Statement 2: Time varies across a longitude, not latitude.
For e.g. when the Prime Meridian of Greenwich has the sun at the highest point in the sky, all the places along this
meridian will have mid-day or noon, whether the place is southwards or northwards.
This is independent of the direction of rotation of the earth. So, 2 is incorrect.
Q Source: Page 15: NCERT 6th Geography: The Earth Our Habitat

87. The Indian Railways plans to introduce Maglev trains that can run at a top speed of 500 km an hour, in a bid to lure
passengers from airlines. Maglev technology harnesses the power of
A. Magnetic fields to propel train at high speed
B. Earths gravitational force
(C) Insights Active Learning. | All rights reserved.

www.insightsias.com

52

TEST - 1

Total Marks :

200

Mark Scored :

110.67

C. Wind in a small tunnel to transfer momentum to the train


D. Solar PV technology to increase the burn rate of fuel in the train engine
User Answer : A
Correct Answer : A
Answer Justification :
Learning: Maglev (derived from magnetic levitation) is a transport method that uses magnetic levitation to move
vehicles without touching the ground.
With maglev, a vehicle travels along a guideway using magnets to create both lift and propulsion, thereby reducing
friction by a great extent and allowing very high speeds.
Maglev trains are in operation in China, Japan, Germany and South Korea. As the trains are propelled by magnetic
forces, friction is eliminated, making transportation free of noise and vibration.
Q Source:
http://www.thehindu.com/news/national/fly-on-rails-global-tenders-floated-to-induct-maglev-trains/article894960
4.ece

88. ScatSat-1 satellite recently launched by ISRO will help in


A. Mineral exploration
B. Forecasting groundwater resources in India
C. Increasing the bandwidth of telecommunication services in India
D. Disaster management
User Answer : D
Correct Answer : D
Answer Justification :
Learning: Currently India is dependent on NASAs ISS-RapidScat for prediction of cyclone forecasting and
weather prediction.
With launch of this satellite India will acquire the capability for more accurate prediction and tracking of
cyclones.
The indigenously developed weather forecasting satellite will be equipped with on board instruments to
monitor sea surface winds and help predict the genesis of cyclones
The primary payload of ScatSat-1 is a scatterometer to keep a watch on the speed and direction of ocean
winds that indicate the formation and strengthening of cyclones.
The satellite will have the capacity to monitor temperature and humidity.
Q Source:
http://www.thehindu.com/todays-paper/tp-national/scatsat-1-to-help-in-predicting-cyclones/article8945917.ece

89. Consider the following statements about the geography of India. Assertion (A): East-West extent of India is greater
than its North-South Extent. Reason (R): India is located entirely in the Northern Hemisphere.
In the context of the above, which of these is correct?
A. A is correct, and R is an appropriate explanation of A.
B. A is correct, but R is not an appropriate explanation of A.
C. A is incorrect, but R is correct.
D. Both A and R are incorrect.
User Answer : B
Correct Answer : C
(C) Insights Active Learning. | All rights reserved.

www.insightsias.com

53

TEST - 1

Total Marks :

200

Mark Scored :

110.67

Answer Justification :
Justification: From south to north, India extends between 84'N and 376'N latitudes. From west to east, India
extends between 687'E and 9725'E longitudes.
So, the north-south extent from Kashmir to Kanyakumari is about 3,200 km. And the east-west extent from
Arunachal Pradesh to Kuchchh is about 2,900 km.
It has nothing to do with which hemisphere India lies in.
Q Source: Page 47: NCERT 6th Geography: The Earth Our Habitat

90. Which of the following generally does NOT come within the functional domain of the Government in India?
A. Maintaining public roads
B. Establishing diplomatic relations with other states
C. Introduction of new railways lines between remote locations
D. Producing goods of daily consumption
User Answer : D
Correct Answer : D
Answer Justification :
Justification: You need to choose most appropriate option out of them.
Option A: NHAI, Ministry of Road Transport etc take care of public roads, national highways in India.
Option B: Ministry of External affairs maintains diplomatic relations with other states.
Option C: It is done by Indian Railways on proposals from the Ministry of Railways.
Option D: Some PSUs do produce goods/commodities of daily consumption like petrol and Diesel, but this is not
the general case. Production of most other articles is left to the private sector, like salt, clothes etc.
Q Source: Page 28: NCERT 6th: Social and Economic Life I

91. Only one side of the moon is visible to us on the earth mainly due to
1. Spin speed of the Moon on its axis which matches with that of earth.
2. Geoid shape and axial tilt of the earth.
3. Atmospheric refraction on earth and absence of atmosphere in space.
Select the correct answer using the codes below.
A. 1 only
B. 1 and 3 only
C. 1 and 2 only
D. 2 and 3 only
User Answer : A
Correct Answer : A
Answer Justification :
Justification: Statement 1: The moon moves around the earth in about 27 days. It takes exactly the same time to
complete one spin. As a result, only one side of the moon is visible to us on the earth. You can watch this
animation for better understanding https://www.youtube.com/watch?v=OZIB_leg75Q
Statement 2: Even if the earth was not tilted, you would be able to see only one side of the moon. The geoid shape
(C) Insights Active Learning. | All rights reserved.

www.insightsias.com

54

TEST - 1

Total Marks :

200

Mark Scored :

110.67

of earth has nothing to do with our view of the Moon. Even if the earth were to become slightly flatter or rounder,
the sight of the moon wont change much.
Statement 3: If at all this was applicable, it should have been applicable to all cosmic bodies, which is not true.
Atmospheric refraction helps planets, stars twinkle and doesnt decide which side of them is visible or invisible on
earth.
Q Source: Page 5: NCERT 6th Geography: The Earth Our Habitat

92. The Food Safety and Standards Authority of India (FSSAI) has been regulating the use of harmful trans fats in
packaged food items. Consider the following about trans fats.
1. They are different from ordinary fats in that they are naturally not found or formed in animal or plant products.
2. They are used by food manufacturers to improve the texture and shelf life of food.
3. Eating them raises the level of low-density lipoprotein (LDL) cholesterol in the blood.
Select the correct answer using the codes below.
A. 2 and 3 only
B. 1 and 2 only
C. 1 and 3 only
D. 1, 2 and 3
User Answer : D
Correct Answer : A
Answer Justification :
Justification: Statement 1: There are two sources of trans fat, also known as trans fatty acids:
Trans fat formed naturally, produced in the gut of some grazing animals as also found in animal products
like meat, milk, and milk products.
Trans fat formed during food processing this type of trans fat is created when hydrogen is added to
vegetable oil (a process called hydrogenation) to make it more solid.
Statement 2: Partially hydrogenated oils are used by food manufacturers to improve the texture, shelf life, and
flavor stability of foods.
Statement 3: Trans fat can be found in many of the same foods as saturated fat. These can include Coffee
creamer, Crackers, cookies, cakes, frozen pies etc.
An elevated LDL (bad) blood cholesterol level can increase your risk of developing cardiovascular disease. HDL
is good blood cholesterol.
Q Source: Improvisation:
http://www.downtoearth.org.in/news/food-safety-agency-extends-implementation-of-trans-fats-standards-by-six-m
onths-55274

93. As per the 73rd Constitutional Amendment Act 1992, "Gram Sabha" means a body consisting of persons
A. Registered in the electoral rolls relating to a village comprised within the area of Panchayat at the village
level
B. Residing in the village or panchayat or contiguous areas by way of naturalization of migration
C. Specified by the Governor by a public notification on the recommendation of the State Election
Commission
D. Chosen so by the Panchayat to address the socio-economic issues of the village
User Answer : A
(C) Insights Active Learning. | All rights reserved.

www.insightsias.com

55

TEST - 1

Total Marks :

200

Mark Scored :

110.67

Correct Answer : A
Answer Justification :
Learning: As per the 73rd Amendment Act 1992, a Gram Sabha may exercise such powers and perform such
functions at the village level as the Legislature of a State may, by law, provide.
There shall be constituted in every State, Panchayats at the village, intermediate and district levels in accordance
with the provisions of this Part.
Subject to the provisions of this Part, the Legislature of a State may, by law, make provisions with respect to the
composition of Panchayats.
We will be covering more question later on the powers and role of the Gram Sabha in a village.
Q Source: Page 43: NCERT 6th: Social and Economic Life I

94. Arrange the following from East to West.


1. Mahadeo Hills
2. Rajmahal Hills
3. Aravalli range
Select the correct answer using the codes below.
A. 213
B. 312
C. 123
D. 231
User Answer : A
Correct Answer : A
Answer Justification :
Justification: The map below is self-explanatory.

Q Source: Map based questions: India

95. Fiscal Monitor report was published in April 2016 by which of these organizations?
(C) Insights Active Learning. | All rights reserved.

www.insightsias.com

56

TEST - 1

Total Marks :

200

Mark Scored :

110.67

A. World Bank
B. World Economic Forum
C. International Monetary Fund
D. Reserve Bank of India
User Answer :
Correct Answer : C
Answer Justification :
Learning: The Fiscal Monitor was launched in 2009 to survey and analyze the latest public finance developments
and other aspects related to it.
It is prepared twice a year by the IMFs Fiscal Affairs Department.
Its projections are based on the same database used for the World Economic Outlook (WEO) and the Global
Financial Stability Report (GFSR).
It is important that the IMF monitors the fiscal situation of its member nations as it acts a lender of last
resort in case of BoP crisis.
Q Source: Current Affairs: IMF Website

96. Scientists have recently developed a new bio-ink that allows 3D printing of complex living tissues that may be used
for surgical implants. The bio-ink contains which of the following?
1. Natural polymer extracted from seaweed
2. Stem cells that have potency and unspecialized states
Which of the above is/are correct?
A. 1 only
B. 2 only
C. Both 1 and 2
D. None
User Answer :
Correct Answer : C
Answer Justification :
Justification: Statement 1: The bio-ink contains two different polymer components: a natural polymer extracted
from seaweed, and a sacrificial synthetic polymer used in the medical industry.
The synthetic polymer causes the bio-ink to change from liquid to solid when the temperature is raised, and the
seaweed polymer provides structural support when the cell nutrients are introduced.
Statement 2: Stem cells differ from other kinds of cells in the body. All stem cells-regardless of their source have three general properties: they are capable of dividing and renewing themselves for long periods; they are
unspecialized; and they can give rise to specialized cell types.
They are crucial for organ transplants as they can regenerate into different types of human organs.
Learning: You can learn about 3-D printing here http://3dprinting.com/what-is-3d-printing/
Q
Source:http://indianexpress.com/article/technology/science/scientists-develop-new-bio-ink-for-3d-printing-28749
08/

(C) Insights Active Learning. | All rights reserved.

www.insightsias.com

57

TEST - 1

Total Marks :

200

Mark Scored :

110.67

97. How is the continental crust different from oceanic crust?


A. Mineral constituents of the continental mass are Silica and alumina, whereas that of oceanic crust is mainly
iron.
B. Continental crust is less dense than oceanic crust.
C. Continental crust has a highly diverse composition of elements in both fused and natural form, whereas
oceanic crust is made from a single element only .
D. All of the above.
User Answer :
Correct Answer : B
Answer Justification :
Justification:Option A: The main mineral constituents of the continental mass are silica and alumina. The
oceanic crust mainly consists of silica and magnesium. So, A is wrong.
It is core that is largely made of iron.
Option B: Both oceanic crust and continental crust are less dense than the mantle, but oceanic crust is denser than
continental crust. This is partly why the continents are at a higher elevation than the ocean floor.
Because continental crust is less dense than oceanic crust it floats higher on the mantle, just like a piece of
Styrofoam floats higher on water than a piece of wood does.
Option C: It is not true, as also mentioned in option A justification.
Q Source: Page 8: Geography NCERT 7th: Our Environment

98. Consider the following statements. Assertion (A): Chief Minister of the State can disqualify a sitting MLA after
obtaining the advice of the State Election Commission. Reason (R): Chief Minister holds the de facto authority in the
State Government.
In the context of the above, which of these is correct?
A. A is correct, and R is an appropriate explanation of A.
B. A is correct, but R is not an appropriate explanation of A.
C. A is incorrect, but R is correct.
D. A is correct, but R is incorrect.
User Answer : C
Correct Answer : C
Answer Justification :
Justification: An MLA can be disqualified on many grounds.
On grounds of defection (to be covered later in tests), the Speaker decides the disqualification.
If he holds any office of profit under the Government of India or a state or an office declared by a law of the
state, he can be disqualified.
If any competent court declares any member to be of unsound mind, then the MLA is disqualified.
Under Article 192 of the Constitution, if any question arises as to whether or not the member of a house of
the legislature of a state has become subject to any of the disqualification criteria, the question shall be cited
to the Governor of the State for decision who will act as per the opinion of the Election Commission (EC).
His decision shall be final and may not be reviewed by any court of law. So, clearly A is wrong.
Q Source: Improvisation: Page 33: NCERT 7th: Social and Economic Life II

(C) Insights Active Learning. | All rights reserved.

www.insightsias.com

58

TEST - 1

Total Marks :

200

Mark Scored :

110.67

99. A Bengali writer, her book titled Amar Jiban is the first known autobiography written by an Indian woman. She
is?
A. Ismat Chugtai
B. Begum Rukhaiya Hussain
C. Deepa Bandopadhyay
D. Rashsundari Devi
User Answer : D
Correct Answer : D
Answer Justification :
Learning: At the age of 60, Rashsundari Devi wrote her autobiography in Bangla in 1860. She and a few others
like (Rukhaiya Hussain) were some the most prominent figures in the literary history of women in colonial India.
Option A: Ismat Chugtai (post-colonial period) was a liberal Muslim writer and considered to be a path-breaking
woman writer of the region.
Option B: Begum Rukhaiya Hussain wrote Sultanas Dream in 1905 to practise her English skills when she was
merely 25 years old. This story imagined a woman called Sultana who reaches a place called Ladyland. Ladyland
is a place where women took similar roles as men.
Q Source: Page 60: NCERT 7th: Social and Economic Life II

100. As per a report commissioned by the Department of Science and Technology (DST), India will be heavily
dependent on China in the coming years to source rare earth minerals for their use in manufacturing sector. What is the
reason behind it?
A. India does not have any reserve of rare-earth minerals.
B. Sensitive imports from Missile Technology Control Regime (MTCR) nations are banned for a non-member
like India.
C. India does not have the necessary technology to extract any rare-earth minerals.
D. China is a leading global supplier for many rare earth mineral resources that have been identified as critical
for India.
User Answer : C
Correct Answer : D
Answer Justification :
Justification:Option A and C: India is endowed with vast mineral resources - it is among the top five nations with
reserves of rare-earth minerals - its potential is untapped.
India currently has a little over 2% share of global output of rare earths. Their production has also increased
post-1995. But, more than 97% supply lies with China.
Option B: These items are not in MTCR lists. MTCR list covers ballistic missile technology.
Option D: China currently accounts for an overwhelming 97% of global production, and it has not been shy of
using this dominance as a bargaining chip against other countries.
Rare earths are vital future resources and India wont like any country to have a monopoly. So, it is
planning to negotiate contracts for future supplies.
Learning: 12 minerals out of 49 that were evaluated as most critical for Indias manufacturing sector by 2030.
These are beryllium, chromium, germanium, limestone, niobium, graphite, rare earths, rhenium, strontium,
(C) Insights Active Learning. | All rights reserved.

www.insightsias.com

59

TEST - 1

Total Marks :

200

Mark Scored :

110.67

tantalum and zirconium.


These elements are critical to many industries of the futurefrom smart phones to hybrid cars to solar
panels.
There is a worldwide scramble to get hold of rare earths. Prices have soared in recent times.
India is 100 per cent import-dependent for seven out of 12 identified critical minerals and does not have any
declared resources for them, except light rare-earths (found along with monazite sands) and beryllium.
You can read more here
http://www.insightsonindia.com/2016/08/05/insights-daily-current-affairs-05-august-2016/
Q Source: As given above

(C) Insights Active Learning. | All rights reserved.

www.insightsias.com

60

Das könnte Ihnen auch gefallen